You are on page 1of 89

1

In the matter of the testate estate of Emil Maurice Bachrach,


deceased. MARY McDONALD BACHRACH,petitioner-appellee,
vs.
SOPHIE SEIFERT and ELISA ELIANOFF, oppositors-appellants.
Ross, Selph, Carrascoso and Janda for appellants.
Delgado and Flores for appellee.

OZAETA, J.:
Is a stock dividend fruit or income, which belongs to the usufructuary, or is
it capital or part of the corpus of the estate, which pertains to the
remainderman? That is the question raised in the appeal.
The deceased E. M. Bachrach, who left no forced heir except his widow Mary
McDonald Bachrach, in his last will and testament made various legacies in
cash and willed the remainder of his estate as follows:
Sixth: It is my will and do herewith bequeath and devise to my
beloved wife Mary McDonald Bachrach for life all the fruits and usufruct
of the remainder of all my estate after payment of the legacies,
bequests, and gifts provided for above; and she may enjoy said
usufruct and use or spend such fruits as she may in any manner wish.
The will further provided that upon the death of Mary McDonald Bachrach,
one-half of the all his estate "shall be divided share and share alike by and
between my legal heirs, to the exclusion of my brothers."
The estate of E. M. Bachrach, as owner of 108,000 shares of stock of the
Atok-Big Wedge Mining Co., Inc., received from the latter 54,000 shares
representing 50 per cent stock dividend on the said 108,000 shares. On June
10, 1948, Mary McDonald Bachrach, as usufructuary or life tenant of the
estate, petitioned the lower court to authorize the Peoples Bank and Trust
Company as administrator of the estate of E. M. Bachrach, to her the said
54,000 share of stock dividend by endorsing and delivering to her the
corresponding certificate of stock, claiming that said dividend, although paid
out in the form of stock, is fruit or income and therefore belonged to her as
usufructuary or life tenant. Sophie Siefert and Elisa Elianoff, legal heirs of
the deceased, opposed said petition on the ground that the stock dividend in
question was not income but formed part of the capital and therefore
belonged not to the usufructuary but to the remainderman. And they have
appealed from the order granting the petition and overruling their objection.
2

While appellants admits that a cash dividend is an income, they contend that
a stock dividend is not, but merely represents an addition to the invested
capital. The so-called Massachusetts rule, which prevails in certain
jurisdictions in the United States, supports appellants' contention . It regards
cash dividends, however large, as income, and stock dividends, however
made, as capital. (Minot vs. Paine, 99 Mass., 101; 96 Am. Dec., 705.) It
holds that a stock dividend is not in any true sense any true sense any
dividend at all since it involves no division or severance from the corporate
assets of the dividend; that it does not distribute property but simply dilutes
the shares as they existed before; and that it takes nothing from the
property of the corporation, and nothing to the interests of the shareholders.
On the other hand, so called Pennsylvania rule, which prevails in various
other jurisdictions in the United States, supports appellee's contention. This
rule declares that all earnings of the corporation made prior to the death of
the testator stockholder belong to the corpus of the estate, and that all
earnings, when declared as dividends in whatever form, made during the
lifetime of the usufructuary or life tenant. (Earp's Appeal, 28 Pa., 368.)
. . . It is clear that testator intent the remaindermen should have only
the corpus of the estate he left in trust, and that all dividends should
go the life tenants. It is true that profits realized are not dividends
until declared by the proper officials of the corporation, but distribution
of profits, however made, in dividends, and the form of the distribution
is immaterial. (In re Thompson's Estate, 262 Pa., 278; 105 Atl. 273,
274.)
In Hite vs. Hite (93 Ky., 257; 20 S. W., 778, 780), the Court of Appeals of
Kentucky, speaking thru its Chief Justice, said:
. . . Where a dividend, although declared in stock, is based upon the
earnings of the company, it is in reality, whether called by one name
or another, the income of the capital invested in it. It is but a mode of
distributing the profit. If it be not income, what is it? If it is, then it is
rightfully and equitably the property of the life tenant. If it be really
profit, then he should have it, whether paid in stock or money. A stock
dividend proper is the issue of new shares paid for by the transfer of a
sum equal to their par value from the profits and loss account to that
representing capital stock; and really a corporation has no right to a
dividend, either in cash or stock, except from its earnings; and a
singular state of case it seems to us, an unreasonable one is
presented if the company, although it rests with it whether it will
declare a dividend, can bind the courts as to the proper ownership of
it, and by the mode of payment substitute its will for that of that of the
3

testator, and favor the life tenants or the remainder-men, as it may
desire. It cannot, in reason, be considered that the testator
contemplated such a result. The law regards substance, and not form,
and such a rule might result not only in a violation of the testator's
intention, but it would give the power to the corporation to beggar the
life tenants, who, in this case, are the wife and children of the testator,
for the benefit of the remainder-men, who may perhaps be unknown
to the testator, being unborn when the will was executed. We are
unwilling to adopt a rule which to us seems so arbitrary, and devoid of
reason and justice. If the dividend be in fact a profit, although
declared in stock, it should be held to be income. It has been so held
in Pennsylvania and many other states, and we think it the correct
rule. Earp's Appeal, 28 Pa. St. 368; Cook, Stocks & S. sec. 554. . . .
We think the Pennsylvania rule is more in accord with our statutory laws
than the Massachusetts rule. Under section 16 of our Corporation Law, no
corporation may make or declare any dividend except from the surplus
profits arising from its business. Any dividend, therefore, whether cash or
stock, represents surplus profits. Article 471 of the Civil Code provides that
the usufructuary shall be entitled to receive all the natural, industrial, and
civil fruits of the property in usufruct. And articles 474 and 475 provide as
follows:
ART. 474. Civil fruits are deemed to accrue day by day, and belong to
the usufructuary in proportion to the time the usufruct may last.
ART. 475. When a usufruct is created on the right to receive an income
or periodical revenue, either in money or fruits, or the interest on
bonds or securities payable to bearer, each matured payment shall be
considered as the proceeds or fruits such right.
When it consists of the enjoyment of the benefits arising from an
interest in an industrial or commercial enterprise, the profits of which
are not distributed at fixed periods, such profits shall have the same
consideration.lawphil.net
In either case they shall be distributed as civil fruits, and shall be
applied in accordance with the rules prescribed by the next preceding
article.
The 108,000 shares of stock are part of the property in usufruct. The 54,000
shares of stock dividend are civil fruits of the original investment. They
represent profits, and the delivery of the certificate of stock covering said
dividend is equivalent to the payment of said profits. Said shares may be
4

sold independently of the original shares, just as the offspring of a domestic
animal may be sold independently of its mother.
The order appealed from, being in accordance with the above-quoted
provisions of the Civil Code, his hereby affirmed, with costs against the
appellants.
Moran, C. J., Paras, Feria, Pablo, Bengzon, Tuason, Montemayor and Reyes,
JJ., concur.
THE BACHRACH MOTOR CO., INC., plaintiff-appellee,
vs.
TALISAY-SILAY MILLING CO., ET AL., defendants-appellees.
THE PHILIPPINE NATIONAL BANK, intervenor-appellant.
Roman J. Lacson for intervenor-appellant.
Mariano Ezpeleta for plaintiff-appellee.
Nolan and Hernaez for defendants-appellees Talisay-Silay Milling Co. and
Cesar Ledesma.
ROMUALDEZ, J.:
This proceeding originated in a complaint filed by the Bachrach Motor Co.,
Inc., against the Talisay-Silay Milling Co., Inc., for the delivery of the
amount P13,850 or promissory notes or other instruments or credit for that
sum payable on June 30, 1930, as bonus in favor of Mariano Lacson
Ledesma; the complaint further prays that the sugar central be ordered to
render an accounting of the amounts it owes Mariano Lacson Ledesma by
way of bonus, dividends, or otherwise, and to pay the plaintiff a sum
sufficient to satisfy the judgment mentioned in the complaint, and that the
sale made by said Mariano Lacson Ledesma be declared null and void.
The Philippine National Bank filed a third party claim alleging a preferential
right to receive any amount which Mariano Lacson Ledesma might be
entitled to from the Talisay-Silay Milling Co. as bonus, because that would be
civil fruits of the land mortgaged to said bank by said debtor for the benefit
of the central referred to, and by virtue of a deed of assignment, and
praying that said central be ordered to delivered directly to the intervening
bank said sum on account of the latter's credit against the aforesaid Mariano
Lacson Ledesma.
The corporation Talisay-Silay Milling Co., Inc., answered the complaint
stating that of Mariano Lacson Ledesma's credit, P7,500 belonged to Cesar
Ledesma because he had purchased it, and praying that it be absolved from
5

the complaint and that the proper party be named so that the remainder
might be delivered.
Cesar Ledesma, in turn, claiming to be the owner by purchase in good faith
an for a reconsideration of the P7,500 which is a part of the credit referred
to above, answered praying that he be absolved from the complaint.
The plaintiff Bachrach Motor Co., Inc., answered the third party claim
alleging that its credit against Mariano Lacson Ledesma was prior and
preferential to that of the intervening bank, and praying that the latter's
complaint be dismissed.
At the trial all the parties agreed to recognize and respect the sale made in
favor of Cesar Ledesma of the P7,500 part of the credit in question, for
which reason the trial court dismissed the complaint and cross-complaint
against Cesar Ledesma authorizing the defendant central to deliver to him
the aforementioned sum of P7,500. And upon conclusion of the hearing, the
court held that the Bachrach Motor Co., Inc., had a preferred right to receive
the amount of P11,076.02 which was Mariano Lacson Ledesma's bonus, and
it ordered the defendant central to deliver said sum to the plaintiff.
The Philippine National Bank appeals, assigning the following alleged errors
as committed by the trial court:
1. In holding that the bonus which the Talisay-Silay Milling Co., Inc.,
bound itself to pay the planters who had mortgaged their land to the
Philippine National Bank to secure the payment of the debt of said
central to said bank is not civil fruits of said land.
2. In not holding that said bonus became subject to the mortgage
executed by the defendant Mariano Lacson Ledesma to the Philippine
National Bank to secure the payment of his personal debt to said bank
when it fell due.
3. In holding that the assignment (Exhibit 9, P.N.B.) of said bonus
made on March 7, 1930, by Mariano Lacson Ledesma to the Philippine
National Bank to be applied to the payment of his debt to said
Philippine National Bank is fraudulent.
4. In holding that the Bachrach Motor Co. Inc., in civil case No. 31597
of the Court of First Instance of Manila levied a valid attachment upon
the bonus in question.
6

5. In admitting and considering the supplementary complaint filed by
the Bachrach Motor Co., Inc., alleging as a cause of action the
attachment of the bonus in question which said Bachrach Motor Co.,
Inc., in civil case No. 31821 of the Court of First Instance of Manila
levied after the filing of the original complaint in this case, and after
Mariano Lacson Ledesma in this case had been declared in default.
6. In holding that the Bachrach Motor Co., Inc., has a preferential right
to receive from the Talisay-Silay Milling Co., Inc., the amount of
P11,076.02 which is in the possession of said corporation as the bonus
to be paid to Mariano Lacson Ledesma, and in ordering the Talisay-
Silay Milling Co., Inc., to deliver said amount to the Bachrach Motor
Co., Inc.
7. In not holding that the Philippine National Bank has a preferential
right to receive from the Talisay-Silay Milling Co., Inc., the amount of
P11,076.02 held by said corporation as Mariano Lacson Ledesma's
bonus, and in not ordering said Talisay-Silay Milling Co., Inc., to
deliver said amount to the Philippine National Bank.
8. In not holding that the amended complaint and the supplementary
complaint of the Bachrach Motor Co., Inc., do not state facts sufficient
to constitute a cause of action in favor of the Bachrach Motor Co., Inc.,
and against the Talisay-Silay Milling Co., Inc., or against the Philippine
National Bank.
The appellant bank bases its preferential right upon the contention that the
bonus in question is civil fruits of the lands which the owners had mortgaged
for the benefit of the central giving the bonus, and that, as civil fruits of said
land, said bonus was assigned by Mariano Lacson Ledesma on March 7,
1930, by virtue of the document Exhibit 9 of said intervening institution,
which admitted in its brief that "if the bonus in question is not civil fruits or
rent which became subject to the mortgage in favor of the Philippine
National Bank when Mariano Lacson Ledesma's personal obligation fell due,
the assignment of March 7, 1930 (Exhibit 9, P.N.B.), is null and void, not
because it is fraudulent, for there was no intent of fraud in executing the
deed, but that the cause or consideration of the assignment was erroneous,
for it was based upon the proposition that the bonus was civil fruits of the
land mortgaged to the Philippine National Bank." (P. 31.)
The fundamental question, then, submitted to our consideration is whether
or not the bonus in question is civil fruits.
7

This is how the bonus came to be granted: On December 22, 1923, the
Talisay-Silay Milling Co., Inc., was indebted to the Philippine National Bank.
To secure the payment of its debt, it succeeded in inducing its planters,
among whom was Mariano Lacson Ledesma, to mortgage their land to the
creditor bank. And in order to compensate those planters for the risk they
were running with their property under the mortgage, the aforesaid central,
by a resolution passed on that same date, i.e., December 22, 1923,
undertook to credit the owners of the plantation thus mortgaged every year
with a sum equal to two per centum of the debt secured according to yearly
balance, the payment of the bonus being made at once, or in part from time
to time, as soon as the central became free of its obligations to the aforesaid
bank, and of those contracted by virtue of the contract of supervision, and
had funds which might be so used, or as soon as it obtained from said bank
authority to make such payment. (Exhibits 5, 6; P.N.B.)
Article 355 of the Civil Code considers three things as civil fruits: First, the
rents of buildings; second, the proceeds from leases of lands; and, third, the
income from perpetual or life annuities, or other similar sources of revenue.
It may be noted that according to the context of the law, the phrase "u otras
analogas" refers only to rent or income, for the
adjectives "otras" and "analogas" agree with the noun "rentas," as do also
the other adjectives "perpetuas" and "vitalicias." That is why we say that by
"civil fruits" the Civil Code understands one of three and only three things, to
wit: the rent of a building, the rent of land, and certain kinds of income.
As the bonus in question is not rent of a building or of land, the only
meaning of "civil fruits" left to be examined is that of "income."
Assuming that in broad juridical sense of the word "income" it might be said
that the bonus in question is "income" under article 355 of the Civil Code, it
is obvious to inquire whether it is derived from the land mortgaged by
Mariano Lacson Ledesma to the appellant bank for the benefit of the central;
for it is not obtained from that land but from something else, it is not civil
fruits of that land, and the bank's contention is untenable.
It is to be noted that the said bonus bears no immediate, but only a remote
accidental relation to the land mentioned, having been granted as
compensation for the risk of having subjected one's land to a lien in favor of
the bank, for the benefit of the entity granting said bonus. If this bonus be
income or civil fruits of anything, it is income arising from said risk, or, if one
chooses, from Mariano Lacson Ledesma's generosity in facing the danger for
the protection of the central, but certainly it is not civil fruits or income from
the mortgaged property, which, as far as this case is concerned, has nothing
to do with it. Hence, the amount of the bonus, according to the resolution of
8

the central granting it, is not based upon the value, importance or any other
circumstance of the mortgaged property, but upon the total value of the
debt thereby secured, according to the annual balance, which is something
quite distinct from and independent of the property referred to.
Finding no merit in this appeal, the judgment appealed from is affirmed,
without express finding as to costs. So ordered.
Johnson, Street, Malcolm, Villamor, Ostrand, Villa-Real, and Imperial,
JJ., concur.
VICENTE STO. DOMINGO BERNARDO, Plaintiff-Appellant, vs. CATALINO
BATACLAN,Defendant-Appellant.
TORIBIO TEODORO, purchaser-appellee.
Pedro de Leon for plaintiff-appellant.
Angel H. Mojica and Francisco Lavides for defendant appellant.
Jose Y. Garde for appellee.
chanrobles virtual law library
LAUREL, J.: chanrobles virtual law library
This is an appeal taken by both the plaintiff and the defendant from the
order of September 26, 1935, hereinabove referred to, of the Court of First
Instance of Cavite in Civil Case No.
2428.chanroblesvirtualawlibrary chanrobles virtual law library
There is no controversy as to the facts. By a contract of sale executed from
Pastor Samonte and others ownership of a parcel of land of about 90
hectares situated in sitio Balayunan, Silang, Cavite. To secure possession of
the land from the vendors the said plaintiff, on July 20, 1929, instituted Civil
Case No. 1935 in the Court of First Instance of Cavite. The trial court found
for the plaintiff in a decision which was affirmed by this Supreme Court on
appeal (G.R. No. 33017).
1
When plaintiff entered upon the premises,
however, he found the defendant herein, Catalino Bataclan, who appears to
have been authorized by former owners, as far back as 1922, to clear the
land and make improvements thereon. As Bataclan was not a party in Case
No. 1935, plaintiff, on June 11, 1931, instituted against him, in the Court of
First Instance of Cavite, Civil Case No. 2428. In this case, plaintiff was
declared owner but the defendant was held to be a possessor in good faith,
entitled to reimbursement in the total sum of P1,642, for work done and
improvements made. The dispositive part of the decision reads:
9

Por las consideraciones expuestas, se declara al demandante Vicente Santo
Domingo Bernardo dueo con derecho a la posesion del terreno que se
describe en la demanda, y al demandado Catalino Bataclan con derecho a
que del demandante le pague la suma de P1,642 por gastos utiles hechos de
buena fe en el terreno, y por el cerco y ponos de coco y abaca existentes en
el mismo, y con derecho, ademas a retener la posesion del terreno hasta
que se le pague dicha cantidad. Al demandante puede optar, en el plazo de
treinta dias, a partir de la fecha en que fuere notificado de la presente, por
pagar esa suma al demandado, haciendo asi suyos el cerco y todas las
plantaciones existentes en el terreno, u obligar al demandado a pagarle el
precio terreno, a razon de trescientos pesos la hectarea. En el caso de que el
demandante optara por que el demandado le pagara el precio del terreno, el
demandado efectuara el pago en el plazo convenientes por las partes o que
sera fijado por el Juzgado. Sin costas.
Both parties appealed to this court (G. R. No. 37319).
2
The decision
appealed from was modified by allowing the defendant to recover
compensation amounting to P2,212 and by reducing the price at which the
plaintiff could require the defendant to purchase the land in question from
P300 to P200 per hectare. Plaintiff was given by this court 30 days from the
date when the decision became final within which to exercise his option,
either to sell the land to the defendant or to buy the improvements from
him. On January 9, 1934, the plaintiff manifested to the lower court his
desire "to require the defendant to pay him the value of the land at the rate
of P200 per hectare or a total price of P18,000 for the whole tract of land."
The defendant informed the lower court that he was unable to pay the land
and, on January 24, 1934, an order was issued giving the plaintiff 30 days
within which to pay the defendant the sum of P2,212 stating that, in the
event of failure to make such payment, the land would be ordered sold at
public auction " Para hacer pago al demandante de la suma de P2,212 y el
remanente despues de deducidos los gastos legales de la venta en publica
subasta sera entregado al demandante." On February 21, 1934, plaintiff
moved to reconsider the foregoing order so that he would have preference
over the defendant in the order of payment. The motion was denied on
March 1, 1934 but on March 16 following the court below, motu
proprio modified its order of January 24, " en el sentido de que el
demandante tiene derecho preferente al importe del terreno no se vendiere
en publica subasta, a razon de P200 por hectares y el remanente, si acaso lo
hubiere se entregara al demandado en pago de la cantidad de P2,212 por la
limpieza del terreno y las mejoras introducidas en el mismo por el citado
demandado." On April 24, 1934, the court below, at the instance of the
plaintiff and without objection on the part of the defendant, ordered the sale
of the land in question at public auction. The land was sold on April 5, 1935
to Toribio Teodoro, the highest bidder, for P8,000. In the certificate of sale
10

issued to said purchaser on the very day of sale, it was stated that the
period of redemption of the land sold was to expire on April 5, 1936. Upon
petition of Toribio Teodoro the court below ordered the provincial sheriff to
issue another certificate not qualified by any equity of redemption. This was
complied with by the sheriff on July 30, 1935. On September 18, 1935,
Teodoro moved that he be placed in possession of the land purchased by
him. The motion was granted by order of September 26, 1935, the
dispositive part of which is as follows:
Por tanto, se ordena al Sheriff Provincial de Cavite ponga a Toribio Teodoro
en posesion del terreno comprado por el en subasta publica y por el cual se
le expidio certificado de venta definitiva, reservando al demandado su
derecho de ejercitar una accion ordinaria para reclamar del demandante la
cantidad de P2,212 a que tiene derecho por la limpieza y mejoras del
terreno y cuya suma, en justicia y equidad, debe ser descontada y deducida
de la suma de P8,000 que ya ha recibido el demandante.
The Civil Code confirms certain time-honored principles of the law of
property. One of these is the principle of accession whereby the owner of
property acquires not only that which it produces but that which is united to
it either naturally or artificially. (Art. 353.) Whatever is built, planted or
sown on the land of another, and the improvements or repairs made
thereon, belong to the owner of the land (art. 358). Where, however, the
planter, builder, or sower has acted in good faith, a conflict of rights arises
between the owners and it becomes necessary to protect the owner of the
improvements without causing injustice to the owner of the land. In view of
the impracticability of creating what Manresa calls a state of "forced
coownership" (vol. 3, 4th ed., p. 213), the law has provided a just and
equitable solution by giving the owner of the land the option to acquire the
improvements after payment of the proper indemnity or to oblige the builder
or planter to pay for the land and the sower to pay the proper rent (art.
361). It is the owner of the land who is allowed to exercise the option
because his right is older and because, by the principle of accession, he is
entitled to the ownership of the accessory thing (3 Manresa, 4th ed., p.
213). In the case before us, the plaintiff, as owner of the land, chose to
require the defendant, as owner of the improvements, to pay for the
land.chanroblesvirtualawlibrary chanrobles virtual law library
The defendant states that he is a possessor in good faith and that the
amount of P2,212 to which he is entitled has not yet been paid to him.
Therefore, he says, he has a right to retain the land in accordance with the
provisions of article 453 of the Civil Code. We do not doubt the validity of
the premises stated. " Considera la ley tan saarada y legitima la deuda, que,
hasta que sea pagada, no consiente que la cosa se restituya all vencedor."
11

(4 Manresa, 4th ed, p., 304.) We find, however, that the defendant has lost
his right of retention. In obedience to the decision of this court in G.R. No.
37319, the plaintiff expressed his desire to require the defendant to pay for
the value of the land. The said defendant could have become owner of both
land and improvements and continued in possession thereof. But he said he
could not pay and the land was sold at public auction to Toribio Teodoro. The
law, as we have already said, requires no more than that the owner of the
land should choose between indemnifying the owner of the improvements or
requiring the latter to pay for the land. When he failed to pay for the land,
the defendant herein lost his right of
retention.chanroblesvirtualawlibrary chanrobles virtual law library
The sale at public auction having been asked by the plaintiff himself (p. 22,
bill of exceptions) and the purchase price of P8,000 received by him from
Toribio Teodoro, we find no reason to justify a rapture of the situation thus
created between them, the defendant-appellant not being entitled, after all,
to recover from the plaintiff the sum of
P2,212.chanroblesvirtualawlibrary chanrobles virtual law library
The judgment of the lower court is accordingly modified by eliminating
therefrom the reservation made in favor of the defendant-appellant to
recover from the plaintiff the sum of P2,212. In all the respects, the same is
affirmed, without pronouncement regarding costs. So
ordered.chanroblesvirtualawlibrary chanrobles virtual law library
Avancea, C.J., Villa-Real, Imperial and Diaz, JJ., concur.
DAMIAN IGNACIO, FRANCISCO IGNACIO and LUIS
IGNACIO Petitioners, vs. ELIAS HILARIO and his wife DIONISIA DRES,
and FELIPE NATIVIDAD, Judge of First Instance of
Pangasinan, Respondents.
Leoncio R. Esliza for petitioners.
Mauricio M. Monta for respondents.
MORAN, C.J.: chanrobles virtual law library
This is a petition for certiorari arising from a case in the Court of First
Instance of Pangasinan between the herein respondents Elias Hilario and his
wife Dionisia Dres as plaintiffs, and the herein petitioners Damian, Francisco
and Luis, surnamed Ignacio, as defendants, concerning the ownership of a
parcel of land, partly rice-land and partly residential. After the trial of the
case, the lower court, presided over by Hon. Alfonso Felix, rendered
judgment holding plaintiffs as the legal owners of the whole property but
12

conceding to defendants the ownership of the houses and granaries built by
them on the residential portion with the rights of a possessor in good faith,
in accordance with article 361 of the Civil Code. The dispositive part of the
decision, hub of this controversy, follows:
Wherefore, judgment is hereby rendered declaring:chanrobles virtual law
library
(1) That the plaintiffs are the owners of the whole property described in
transfer certificate of title No. 12872 (Exhibit A) issued in their name, and
entitled to the possession of the same;chanrobles virtual law library
(2) That the defendants are entitled to hold the position of the residential lot
until after they are paid the actual market value of their houses and
granaries erected thereon, unless the plaintiffs prefer to sell them said
residential lot, in which case defendants shall pay the plaintiffs the
proportionate value of said residential lot taking as a basis the price paid for
the whole land according to Exhibit B; and chanrobles virtual law library
(3) That upon defendant's failure to purchase the residential lot in question,
said defendants shall remove their houses and granaries after this decision
becomes final and within the period of sixty (60) days from the date that the
court is informed in writing of the attitude of the parties in this
respect.chanroblesvirtualawlibrary chanrobles virtual law library
No pronouncement is made as to damages and
costs.chanroblesvirtualawlibrary chanrobles virtual law library
Once this decision becomes final, the plaintiffs and defendants may appear
again before this court for the purpose of determining their respective rights
under article 361 of the Civil Code, if they cannot come to an extra-judicial
settlement with regard to said rights.
Subsequently, in a motion filed in the same Court of First Instance but now
presided over by the herein respondent Judge Hon. Felipe Natividad, the
plaintiffs prayed for an order of execution alleging that since they chose
neither to pay defendants for the buildings nor to sell to them the residential
lot, said defendants should be ordered to remove the structure at their own
expense and to restore plaintiffs in the possession of said lot. Defendants
objected to this motion which, after hearing, was granted by Judge
Natividad. Hence, this petition by defendants praying for (a) a restraint and
annulment of the order of execution issued by Judge Natividad; (b) an order
to compel plaintiffs to pay them the sum of P2,000 for the buildings, or sell
to them the residential lot for P45; or (c), a rehearing of the case for a
13

determination of the rights of the parties upon failure of extra-judicial
settlement.chanroblesvirtualawlibrary chanrobles virtual law library
The judgment rendered by Judge Felix is founded on articles 361 and 453 of
the Civil Code which are as follows:
ART. 361. The owner of land on which anything has been built, sown or
planted in good faith, shall have the right to appropriate as his own the
work, sowing or planting, after the payment of the indemnity stated in
articles 453 and 454, or to oblige the one who built or planted to pay the
price of the land, and the one who sowed, the proper
rent.chanroblesvirtualawlibrary chanrobles virtual law library
ART. 453. Necessary expenses shall be refunded to every possessor; but
only the possessor in good faith may retain the thing until such expenses are
made good to him.chanroblesvirtualawlibrary chanrobles virtual law library
Useful expenses shall be refunded to the possessor in good faith with the
same right of retention, the person who has defeated him in the possession
having the option of refunding the amount of the expenses or paying the
increase in value which the thing may have acquired in consequence thereof.
The owner of the building erected in good faith on a land owned by another,
is entitled to retain the possession of the land until he is paid the value of his
building, under article 453. The owner of the land, upon the other hand, has
the option, under article 361, either to pay for the building or to sell his land
to the owner of the building. But he cannot, as respondents here did, refuse
both to pay for the building and to sell the land and compel the owner of the
building to remove it from the land where it is erected. He is entitled to such
remotion only when, after having chosen to sell his land, the other party fails
to pay for the same. But this is not the case before
us.chanroblesvirtualawlibrary chanrobles virtual law library
We hold, therefore, that the order of Judge Natividad compelling defendants-
petitioners to remove their buildings from the land belonging to plaintiffs-
respondents only because the latter chose neither to pay for such buildings
not to sell the land, is null and void, for it amends substantially the
judgment sought to be executed and is, furthermore, offensive to articles
361 and 453 of the Civil Code.chanroblesvirtualawlibrary chanrobles virtual
law library
There is, however, in the decision of Judge Felix a question of procedure
which calls for the clarification, to avoid uncertainty and delay in the
disposition of cases. In that decision, the rights of both parties are well
14

defined under articles 361 and 453 of the Civil Code, but it fails to determine
the value of the buildings and of the lot where they are erected as well as
the periods of time within which the option may be exercised and payment
should be made, these particulars having been left for determination
apparently after the judgment has become final. This procedure is
erroneous, for after the judgment has become final, no additions can be
made thereto and nothing can be done therewith except its execution. And
execution cannot be had, the sheriff being ignorant as to how, for how
much, and within what time may the option be exercised, and certainly no
authority is vested in him to settle these matters which involve exercise of
judicial discretion. Thus the judgment rendered by Judge Felix has never
become final, it having left matters to be settled for its completion in a
subsequent proceeding, matters which remained unsettled up to the time
the petition is filed in the instant case.chanroblesvirtualawlibrary chanrobles
virtual law library
For all the foregoing, the writ of execution issued by Judge Natividad is
hereby set aside and the lower court ordered to hold a hearing in the
principal case wherein it must determine the prices of the buildings and of
the residential lot where they are erected, as well as the period of time
within which the plaintiffs-respondents may exercise their option either to
pay for the buildings or to sell their land, and, in the last instance, the period
of time within which the defendants-petitioners may pay for the land, all
these periods to be counted from the date the judgment becomes executory
or unappealable. After such hearing, the court shall render a final judgment
according to the evidence presented by the
parties.chanroblesvirtualawlibrary chanrobles virtual law library
The costs shall be paid by plaintiffs-
respondents.chanroblesvirtualawlibrary chanrobles virtual law library
Ozaeta, Paras, Jaranilla, Feria, De Joya, Pablo, Perfecto, Hilado, Bengzon and
Briones, JJ., concur.
LEONILA SARMINETO, petitioner,
vs.
HON. ENRIQUE A. AGANA, District Judge, Court of First Instance of
Rizal, Seventh Judicial District, Branch XXVIII, Pasay City, and
SPOUSES ERNESTO VALENTINO and REBECCA LORENZO-
VALENTINO,respondents.
Mercedes M. Respicio for petitioner.
Romulo R. Bobadilla for private respondents.
15


MELENCIO-HERRERA, J.:+.wph!1
This Petition for certiorari questions a March 29, 1979 Decision rendered by
the then Court of First Instance of Pasay City. The Decision was one made
on memoranda, pursuant to the provisions of RA 6031, and it modified, on
October 17, 1977, a judgment of the then Municipal Court of Paranaque,
Rizal, in an Ejectment suit instituted by herein petitioner Leonila SARMIENTO
against private respondents, the spouses ERNESTO Valentino and Rebecca
Lorenzo. For the facts, therefore, we have to look to the evidence presented
by the parties at the original level.
It appears that while ERNESTO was still courting his wife, the latter's mother
had told him the couple could build a RESIDENTIAL HOUSE on a lot of 145
sq. ms., being Lot D of a subdivision in Paranaque (the LAND, for short). In
1967, ERNESTO did construct a RESIDENTIAL HOUSE on the LAND at a cost
of P8,000.00 to P10,000.00. It was probably assumed that the wife's mother
was the owner of the LAND and that, eventually, it would somehow be
transferred to the spouses.
It subsequently turned out that the LAND had been titled in the name of Mr.
& Mrs. Jose C. Santo, Jr. who, on September 7 , 1974, sold the same to
petitioner SARMIENTO. The following January 6, 1975, SARMIENTO asked
ERNESTO and wife to vacate and, on April 21, 1975, filed an Ejectment suit
against them. In the evidentiary hearings before the Municipal Court,
SARMIENTO submitted the deed of sale of the LAND in her favor, which
showed the price to be P15,000.00. On the other hand, ERNESTO testified
that the then cost of the RESIDENTIAL HOUSE would be from P30,000.00 to
P40,000.00. The figures were not questioned by SARMIENTO.
The Municipal Court found that private respondents had built the
RESIDENTIAL HOUSE in good faith, and, disregarding the testimony of
ERNESTO, that it had a value of P20,000.00. It then ordered ERNESTO and
wife to vacate the LAND after SARMIENTO has paid them the mentioned sum
of P20,000.00.
The Ejectment suit was elevated to the Court of First Instance of Pasay
where, after the submission of memoranda, said Court rendered a modifying
Decision under Article 448 of the Civil Code. SARMIENTO was required,
within 60 days, to exercise the option to reimburse ERNESTO and wife the
sum of 40,000.00 as the value of the RESIDENTIAL HOUSE, or the option to
allow them to purchase the LAND for P25,000.00. SARMIENTO did not
exercise any of the two options within the indicated period, and ERNESTO
16

was then allowed to deposit the sum of P25,000.00 with the Court as the
purchase price for the LAND. This is the hub of the controversy. SARMIENTO
then instituted the instant certiorari proceedings.
We agree that ERNESTO and wife were builders in good faith in view of the
peculiar circumstances under which they had constructed the RESIDENTIAL
HOUSE. As far as they knew, the LAND was owned by ERNESTO's mother-in-
law who, having stated they could build on the property, could reasonably be
expected to later on give them the LAND.
In regards to builders in good faith, Article 448 of the Code
provides:t.hqw
ART. 448. The owner of the land on which anything has been
built, sown or planted in good faith,
shall have the right
to appropriate as his own the works, sowing or planting, after
payment of the indemnity provided for in articles 546 and 548,
or
to oblige the one who built or planted to pay the price of the
land, and the one who sowed, the proper rent.
However, the builder or planter cannot be obliged to buy the
land if its value is considerably more than that of the building or
trees. In such case, he shall pay reasonable rent, if the owner of
the land does not choose to appropriate the building or trees
after proper indemnity. The parties shall agree upon the terms of
the lease and in case of disagreement, the court shall fix the
terms thereof. (Paragraphing supplied)
The value of the LAND, purchased for P15,000.00 on September 7, 1974,
could not have been very much more than that amount during the following
January when ERNESTO and wife were asked to vacate. However, ERNESTO
and wife have not questioned the P25,000.00 valuation determined by the
Court of First Instance.
In regards to the valuation of the RESIDENTIAL HOUSE, the only evidence
presented was the testimony of ERNESTO that its worth at the time of the
trial should be from P30,000.00 to P40,000.00. The Municipal Court chose to
assess its value at P20,000.00, or below the minimum testified by ERNESTO,
while the Court of First Instance chose the maximum of P40,000.00. In the
17

latter case, it cannot be said that the Court of First Instance had abused its
discretion.
The challenged decision of respondent Court, based on valuations of
P25,000.00 for the LAND and P40,000.00 for the RESIDENTIAL HOUSE,
cannot be viewed as not supported by the evidence. The provision for the
exercise by petitioner SARMIENTO of either the option to indemnify private
respondents in the amount of P40,000.00, or the option to allow private
respondents to purchase the LAND at P25,000.00, in our opinion, was a
correct decision.t.hqw
The owner of the building erected in good faith on a land owned
by another, is entitled to retain the possession of the land until
he is paid the value of his building, under article 453 (now Article
546). The owner, of the land. upon, the other hand, has the
option, under article 361 (now Article 448), either to pay for the
building or to sell his land to the owner of the building. But he
cannot, as respondents here did, refuse both to pay for the
building and to sell the land and compel the owner of the
building to remove it from the land where it is erected. He is
entitled to such remotion only when, after having chosen to sell
his land, the other party fails to pay for the same. (Emphasis
ours)
We hold, therefore, that the order of Judge Natividad compelling
defendants-petitioners to remove their buildings from the land
belonging to plaintiffs-respondents only because the latter chose
neither to pay for such buildings nor to sell the land, is null and
void, for it amends substantially the judgment sought to be
executed and is, furthermore, offensive to articles 361 (now
Article 448) and 453 (now Article 546) of the Civil Code. (Ignacio
vs. Hilario, 76 Phil. 605, 608 [1946]).
WHEREFORE, the Petition for Certiorari is hereby ordered dismissed, without
pronouncement as to costs.
SO ORDERED.1wph1.t
FRANCISCO DEPRA, plaintiff-appellee,
vs.
AGUSTIN DUMLAO, defendant-appellant.
Roberto D. Dineros for plaintiff-appellee.
18

Veil D. Hechanova for defendant-appellant.

MELENCIO-HERRERA, J.:
This is an appeal from the Order of the former Court of First Instance of
Iloilo to the then Court of Appeals, which the latter certified to this instance
as involving pure questions of law
Plaintiff-appellee, Francisco Depra, is the owner of a parcel of land registered
under Transfer Certificate of Title No. T3087, known as Lot No. 685, situated
in the municipality of Dumangas, Iloilo, with an area of approximately 8,870
square meters. Agustin Dumlao, defendant-appellant, owns an adjoining lot,
designated as Lot No. 683, with an approximate area of 231 sq. ms.
Sometime in 1972, when DUMLAO constructed his house on his lot, the
kitchen thereof had encroached on an area of thirty four (34) square meters
of DEPRA's property, After the encroachment was discovered in a relocation
survey of DEPRA's lot made on November 2,1972, his mother, Beatriz Depra
after writing a demand letter asking DUMLAO to move back from his
encroachment, filed an action for Unlawful Detainer on February 6,1973
against DUMLAO in the Municipal Court of of Dumangas, docketed as Civil
Case No 1, Said complaint was later amended to include DEPRA as a party
plain. plaintiff.
After trial, the Municipal Court found that DUMLAO was a builder in good
faith, and applying Article 448 of the Civil Code, rendered judgment on
September 29, 1973, the dispositive portion of which reads:
Ordering that a forced lease is created between the parties with
the plaintiffs, as lessors, and the defendants as lessees, over the
disputed portion with an area of thirty four (34) square meters,
the rent to be paid is five (P5.00) pesos a month, payable by the
lessee to the lessors within the first five (5) days of the month
the rent is due; and the lease shall commence on the day that
this decision shall have become final.
From the foregoing judgment, neither party appeal so that, ff it were a valid
judgment, it would have ordinarily lapsed into finality, but even then, DEPRA
did not accept payment of rentals so that DUMLAO deposited such rentals
with the Municipal Court.
19

On July 15,1974, DEPRA filed a Complaint for Quieting of Title against
DUMLAO before the then Court of First Instance of Iloilo, Branch IV (Trial
Court), involving the very same 34 square meters, which was the bone of
contention in the Municipal Court. DUMLAO, in his Answer, admitted the
encroachment but alleged, in the main, that the present suit is barred by res
judicata by virtue of the Decision of the Municipal Court, which had become
final and executory.
After the case had been set for pre-trial, the parties submitted a Joint Motion
for Judgment based on the Stipulation of Facts attached thereto. Premised
thereon, the Trial Court on October 31, 1974, issued the assailed Order,
decreeing:
WHEREFORE, the Court finds and so holds that the thirty four
(34) square meters subject of this litigation is part and parcel of
Lot 685 of the Cadastral Survey of Dumangas of which the
plaintiff is owner as evidenced by Transfer Certificate of Title No.
3087 and such plaintiff is entitled to possess the same.
Without pronouncement as to costs.
SO ORDERED.
Rebutting the argument of res judicata relied upon by DUMLAO, DEPRA
claims that the Decision of the Municipal Court was null and void ab
initio because its jurisdiction is limited to the sole issue of possession,
whereas decisions affecting lease, which is an encumbrance on real
property, may only be rendered by Courts of First Instance.
Addressing out selves to the issue of validity of the Decision of the Municipal
Court, we hold the same to be null and void. The judgment in a detainer
case is effective in respect of possession only (Sec. 7, Rule 70, Rules of
Court).
1
The Municipal Court over-stepped its bounds when it imposed upon
the parties a situation of "forced lease", which like "forced co-ownership" is
not favored in law. Furthermore, a lease is an interest in real property,
jurisdiction over which belongs to Courts of First Instance (now Regional
Trial Courts) (Sec. 44(b), Judiciary Act of 1948;
2
Sec. 19 (2) Batas
Pambansa Blg. 129).
3
Since the Municipal Court, acted without jurisdiction,
its Decision was null and void and cannot operate as res judicata to the
subject complaint for Queting of Title. Besides, even if the Decision were
valid, the rule on res judicata would not apply due to difference in cause of
action. In the Municipal Court, the cause of action was the deprivation of
possession, while in the action to quiet title, the cause of action was based
on ownership. Furthermore, Sec. 7, Rule 70 of the Rules of Court explicitly
20

provides that judgment in a detainer case "shall not bar an action between
the same parties respecting title to the land. "
4

Conceded in the Stipulation of Facts between the parties is that DUMLAO
was a builder in good faith. Thus,
8. That the subject matter in the unlawful detainer case, Civil
Case No. 1, before the Municipal Court of Dumangas, Iloilo
involves the same subject matter in the present case, the Thirty-
four (34) square meters portion of land and built thereon in good
faith is a portion of defendant's kitchen and has been in the
possession of the defendant since 1952 continuously up to the
present; ... (Emphasis ours)
Consistent with the principle that our Court system, like any other, must be
a dispute resolving mechanism, we accord legal effect to the agreement of
the parties, within the context of their mutual concession and stipulation.
They have, thereby, chosen a legal formula to resolve their dispute to appeal
ply to DUMLAO the rights of a "builder in good faith" and to DEPRA those of
a "landowner in good faith" as prescribed in Article 448. Hence, we shall
refrain from further examining whether the factual situations of DUMLAO and
DEPRA conform to the juridical positions respectively defined by law, for a
"builder in good faith" under Article 448, a "possessor in good faith" under
Article 526 and a "landowner in good faith' under Article 448.
In regards to builders in good faith, Article 448 of the Civil Code provides:
ART. 448. The owner of the land on which anything has been built sown or
planted in good faith,
shall have the right
to appropriate as his own the works, sowing or planting, after
payment of the indemnity provided for in articles 546 and 548,
or
to oblige the one who built or planted to pay the price of the
land, and the one who sowed, the proper rent.
However, the builder or planter cannot be obliged to buy the
land if its value is considerably more than that of the building or
trees. In such case, he shall pay reasonable rent, if the owner of
the land does not choose to appropriate the building or trees
after proper indemnity. The parties shall agree upon the terms of
21

the lease and in case of disagreement, the court shall fix the
terms thereof (Paragraphing supplied)
Pursuant to the foregoing provision, DEPRA has the option either to pay for
the encroaching part of DUMLAO's kitchen, or to sell the encroached 34
square meters of his lot to DUMLAO. He cannot refuse to pay for the
encroaching part of the building, and to sell the encroached part of his
land,
5
as he had manifested before the Municipal Court. But that
manifestation is not binding because it was made in a void proceeding.
However, the good faith of DUMLAO is part of the Stipulation of Facts in the
Court of First Instance. It was thus error for the Trial Court to have ruled
that DEPRA is "entitled to possession," without more, of the disputed portion
implying thereby that he is entitled to have the kitchen removed. He is
entitled to such removal only when, after having chosen to sell his
encroached land, DUMLAO fails to pay for the same.
6
In this case, DUMLAO
had expressed his willingness to pay for the land, but DEPRA refused to sell.
The owner of the building erected in good faith on a land owned
by another, is entitled to retain the possession of the land until
he is paid the value of his building, under article 453 (now Article
546). The owner of the land, upon the other hand, has the
option, under article 361 (now Article 448), either to pay for the
building or to sell his land to the owner of the building. But he
cannot as respondents here did refuse both to pay for the
building and to sell the land and compel the owner of the
building to remove it from the land where it erected. He is
entitled to such remotion only when, after having chosen to sell
his land. the other party fails to pay for the same (italics ours).
We hold, therefore, that the order of Judge Natividad compelling
defendants-petitioners to remove their buildings from the land
belonging to plaintiffs-respondents only because the latter chose
neither to pay for such buildings nor to sell the land, is null and
void, for it amends substantially the judgment sought to be
executed and is. furthermore, offensive to articles 361 (now
Article 448) and 453 (now Article 546) of the Civil Code. (Ignacio
vs. Hilario, 76 Phil. 605, 608[1946]).
A word anent the philosophy behind Article 448 of the Civil rode.
The original provision was found in Article 361 of the Spanish Civil Code;
which provided:
22

ART. 361. The owner of land on which anything has been built,
sown or planted in good faith, shall have the right to appropriate
as his own the work, sowing or planting, after the payment of
the indemnity stated in Articles 453 and 454, or to oblige the
one who built or planted to pay the price of the land, and the
one who sowed, the proper rent.
As will be seen, the Article favors the owner of the land, by giving him one of
the two options mentioned in the Article. Some commentators have
questioned the preference in favor of the owner of the land, but Manresa's
opinion is that the Article is just and fair.
. . . es justa la facultad que el codigo da al dueno del suelo en el
articulo 361, en el caso de edificacion o plantacion? Algunos
comentaristas la conceptuan injusta, y como un extraordinario
privilegio en favor de la propiedad territorial. Entienden que
impone el Codigo una pena al poseedor de buena fe y como
advierte uno de los comentaristas aludidos 'no se ve claro el por
que de tal pena . . . al obligar al que obro de buena fe a
quedarse con el edificio o plantacion, previo el pago del terreno
que ocupa, porque si bien es verdad que cuando edifico o planto
demostro con este hecho, que queria para si el edificio o plantio
tambien lo es que el que edifico o planto de buena fe lo hizo en
la erronea inteligencia de creerse dueno del terreno Posible es
que, de saber lo contrario, y de tener noticia de que habia que
comprar y pagar el terreno, no se hubiera decidido a plantar ni a
edificar. La ley obligandole a hacerlo fuerza su voluntad, y la
fuerza por un hecho inocente de que no debe ser responsable'.
Asi podra suceder pero la realidad es que con ese hecho
voluntario, aunque sea inocente, se ha enriquecido
torticeramente con perjuicio de otro a quien es justo
indemnizarle,
En nuestra opinion, el Codigo ha resuelto el conflicto de la
manera mas justa y equitativa y respetando en lo possible el
principio que para la accesion se establece en el art. 358.
7

Our own Code Commission must have taken account of the objections to
Article 361 of the Spanish Civil Code. Hence, the Commission provided a
modification thereof, and Article 448 of our Code has been made to provide:
ART. 448. The owner of the land on which anything has been
built, sown or planted in good faith, shall have the right to
appropriate as his own the works, sowing or planting, after
23

payment of the indemnity provided for in articles 546 and 548,
or to oblige the one who built or planted to pay the price of the
land, and the one who sowed, the proper rent. However, the
builder or planter cannot be obliged to buy the land if its value is
considerably more than that of the building or trees. In such
case, he shall pay reasonable rent, if the owner of the land does
not choose to appropriate the building or trees after proper
indemnity. The parties shall agree upon the terms of the lease
and in case of disagreement, the court shall fix the terms
thereof.
Additional benefits were extended to the builder but the landowner retained
his options.
The fairness of the rules in Article 448 has also been explained as follows:
Where the builder, planter or sower has acted in good faith, a
conflict of rights arises between the owners, and it becomes
necessary to protect the owner of the improvements without
causing injustice to the owner of the land. In view of the
impracticability of creating a state of forced co-ownership, the
law has provided a just solution by giving the owner of the land
the option to acquire the improvements after payment of the
proper indemnity, or to oblige the builder or planter to pay for
the land and the sower to pay for the proper rent. It is the owner
of the land who is authorized to exercise the option, because his
right is older, and because, by the principle of accession, he is
entitled to the ownership of the accessory thing. (3 Manresa
213; Bernardo vs. Bataclan, 37 Off. Gaz. 1382; Co Tao vs. Chan
Chico, G.R. No. 49167, April 30, 1949; Article applied: see
Cabral, et al vs. Ibanez [S.C.] 52 Off. Gaz. 217; Marfori vs.
Velasco, [C.A.] 52 Off. Gaz. 2050).
8

WHEREFORE, the judgment of the trial Court is hereby set aside, and this
case is hereby ordered remanded to the Regional Trial Court of Iloilo for
further proceedings consistent with Articles 448 and 546 of the Civil Code,
as follows:
1. The trial Court shall determine
a) the present fair price of DEPRA's 34 square meter area of
land;
24

b) the amount of the expenses spent by DUMLAO for the building
of the kitchen;
c) the increase in value ("plus value") which the said area of 34
square meters may have acquired by reason thereof, and
d) whether the value of said area of land is considerably more
than that of the kitchen built thereon.
2. After said amounts shall have been determined by competent evidence,
the Regional, Trial Court shall render judgment, as follows:
a) The trial Court shall grant DEPRA a period of fifteen (15) days
within which to exercise his option under the law (Article 448,
Civil Code), whether to appropriate the kitchen as his own by
paying to DUMLAO either the amount of tile expenses spent by
DUMLAO f or the building of the kitchen, or the increase in value
("plus value") which the said area of 34 square meters may have
acquired by reason thereof, or to oblige DUMLAO to pay the
price of said area. The amounts to be respectively paid by
DUMLAO and DEPRA, in accordance with the option thus
exercised by written notice of the other party and to the Court,
shall be paid by the obligor within fifteen (15) days from such
notice of the option by tendering the amount to the Court in
favor of the party entitled to receive it;
b) The trial Court shall further order that if DEPRA exercises the
option to oblige DUMLAO to pay the price of the land but the
latter rejects such purchase because, as found by the trial Court,
the value of the land is considerably more than that of the
kitchen, DUMLAO shall give written notice of such rejection to
DEPRA and to the Court within fifteen (15) days from notice of
DEPRA's option to sell the land. In that event, the parties shall
be given a period of fifteen (15) days from such notice of
rejection within which to agree upon the terms of the lease, and
give the Court formal written notice of such agreement and its
provisos. If no agreement is reached by the parties, the trial
Court, within fifteen (15) days from and after the termination of
the said period fixed for negotiation, shall then fix the terms of
the lease, provided that the monthly rental to be fixed by the
Court shall not be less than Ten Pesos (P10.00) per month,
payable within the first five (5) days of each calendar month.
The period for the forced lease shall not be more than two (2)
years, counted from the finality of the judgment, considering the
25

long period of time since 1952 that DUMLAO has occupied the
subject area. The rental thus fixed shall be increased by ten
percent (10%) for the second year of the forced lease. DUMLAO
shall not make any further constructions or improvements on the
kitchen. Upon expiration of the two-year period, or upon default
by DUMLAO in the payment of rentals for two (2) consecutive
months, DEPRA shall be entitled to terminate the forced lease, to
recover his land, and to have the kitchen removed by DUMLAO
or at the latter's expense. The rentals herein provided shall be
tendered by DUMLAO to the Court for payment to DEPRA, and
such tender shall constitute evidence of whether or not
compliance was made within the period fixed by the Court.
c) In any event, DUMLAO shall pay DEPRA an amount computed
at Ten Pesos (P10.00) per month as reasonable compensation
for the occupancy of DEPRA's land for the period counted from
1952, the year DUMLAO occupied the subject area, up to the
commencement date of the forced lease referred to in the
preceding paragraph;
d) The periods to be fixed by the trial Court in its Precision shall
be inextendible, and upon failure of the party obliged to tender
to the trial Court the amount due to the obligee, the party
entitled to such payment shall be entitled to an order of
execution for the enforcement of payment of the amount due
and for compliance with such other acts as may be required by
the prestation due the obligee.
No costs,
SO ORDERED.
TECNOGAS PHILIPPINES MANUFACTURING
CORPORATION, petitioner, vs.COURT OF APPEALS (FORMER
SPECIAL SEVENTEENTH DIVISION) and EDUARDO
UY, respondents.
D E C I S I O N
PANGANIBAN, J.:
The parties in this case are owners of adjoining lots in Paraaque, Metro
Manila. It was discovered in a survey that a portion of a building of
petitioner, which was presumably constructed by its predecessor-in-interest,
encroached on a portion of the lot owned by private respondent. What are
26

the rights and obligations of the parties? Is petitioner considered a builder
in bad faith because, as held by respondent Court, he is presumed to know
the metes and bounds of his property as described in his certificate of
title? Does petitioner succeed into the good faith or bad faith of his
predecessor-in-interest which presumably constructed the building?
These are the questions raised in the petition for review of the
Decision
[1]
dated August 28, 1992, in CA-G.R. CV No. 28293 of respondent
Court
[2]
where the disposition reads:
[3]

WHEREFORE, premises considered, the Decision of the Regional Trial Court
is hereby reversed and set aside and another one entered -
1. Dismissing the complaint for lack of cause of action;
2. Ordering Tecnogas to pay the sum of P2,000.00 per month as reasonable
rental from October 4, 1979 until appellee vacates the land;
3. To remove the structures and surrounding walls on the encroached area;
4. Ordering appellee to pay the value of the land occupied by the two-storey
building;
5. Ordering appellee to pay the sum of P20,000.00 for and as attorneys
fees;
6. Costs against appellee.
Acting on the motions for reconsideration of both petitioner and private
respondent, respondent Court ordered the deletion of paragraph 4 of the
dispositive portion in an Amended Decision dated February 9, 1993, as
follows:
[4]

WHEREFORE, premises considered, our decision of August 28, 1992 is
hereby modified deleting paragraph 4 of the dispositive portion of our
decision which reads:
4. Ordering appellee to pay the value of the land occupied by the
two-storey building.
The motion for reconsideration of appellee is hereby DENIED for lack of
merit.
The foregoing Amended Decision is also challenged in the instant
petition.
27

The Facts
The facts are not disputed. Respondent Court merely reproduced the
factual findings of the trial court, as follows:
[5]

That plaintiff (herein petitioner) which is a corporation duly organized and
existing under and by virtue of Philippine laws is the registered owner of a
parcel of land situated in Barrio San Dionisio, Paraaque, Metro Manila
known as Lot 4331-A (should be 4531-A) of Lot 4531 of the Cadastral
Survey of Paraaque, Metro Manila, covered by Transfer Certificate of Title
No. 409316 of the Registry of Deeds of the Province of Rizal; that said land
was purchased by plaintiff from Pariz Industries, Inc. in 1970, together with
all the buildings and improvements including the wall existing thereon; that
the defendant (herein private respondent) is the registered owner of a parcel
of land known as Lot No. 4531-B of Lot 4531 of the Cadastral Survey of
Paraaque, LRC (GLRO) Rec. No. 19645 covered by Transfer Certificate of
Title No. 279838, of the Registry of Deeds for the Province of Rizal; that said
land which adjoins plaintiffs land was purchased by defendant from a certain
Enrile Antonio also in 1970; that in 1971, defendant purchased another lot
also adjoining plaintiffs land from a certain Miguel Rodriguez and the same
was registered in defendants name under Transfer Certificate of Title No.
31390, of the Registry of Deeds for the Province of Rizal; that portions of the
buildings and wall bought by plaintiff together with the land from Pariz
Industries are occupying a portion of defendants adjoining land; that upon
learning of the encroachment or occupation by its buildings and wall of a
portion of defendants land, plaintiff offered to buy from defendant that
particular portion of defendants land occupied by portions of its buildings
and wall with an area of 770 square meters, more or less, but defendant,
however, refused the offer. In 1973, the parties entered into a private
agreement before a certain Col. Rosales in Malacaang, wherein plaintiff
agreed to demolish the wall at the back portion of its land thus giving to
defendant possession of a portion of his land previously enclosed by
plaintiffs wall; that defendant later filed a complaint before the office of
Municipal Engineer of Paraaque, Metro Manila as well as before the Office of
the Provincial Fiscal of Rizal against plaintiff in connection with the
encroachment or occupation by plaintiffs buildings and walls of a portion of
its land but said complaint did not prosper; that defendant dug or caused to
be dug a canal along plaintiffs wall, a portion of which collapsed in June,
1980, and led to the filing by plaintiff of the supplemental complaint in the
above-entitled case and a separate criminal complaint for malicious mischief
against defendant and his wife which ultimately resulted into the conviction
in court of defendants wife for the crime of malicious mischief; that while
trial of the case was in progress, plaintiff filed in Court a formal proposal for
28

settlement of the case but said proposal, however, was ignored by
defendant.
After trial on the merits, the Regional Trial Court
[6]
of Pasay City, Branch
117, in Civil Case No. PQ-7631-P, rendered a decision dated December 4,
1989 in favor of petitioner who was the plaintiff therein. The dispositive
portion reads:
[7]

WHEREFORE, judgment is hereby rendered in favor of plaintiff and against
defendant and ordering the latter to sell to plaintiff that portion of land
owned by him and occupied by portions of plaintiffs buildings and wall at the
price of P2,000.00 per square meter and to pay the former:
1. The sum of P44,000.00 to compensate for the losses in materials and
properties incurred by plaintiff through thievery as a result of the destruction
of its wall;
2. The sum of P7,500.00 as and by way of attorneys fees; and
3. The costs of this suit.
Appeal was duly interposed with respondent Court, which as previously
stated, reversed and set aside the decision of the Regional Trial Court and
rendered the assailed Decision and Amended Decision. Hence, this recourse
under Rule 45 of the Rules of Court.
The Issues
The petition raises the following issues:
[8]

(A)
Whether or not the respondent Court of Appeals erred in holding the
petitioner a builder in bad faith because it is presumed to know the
metes and bounds of his property.
(B)
Whether or not the respondent Court of Appeals erred when it used the
amicable settlement between the petitioner and the private respondent,
where both parties agreed to the demolition of the rear portion of the
fence, as estoppel amounting to recognition by petitioner of respondents
right over his property including the portions of the land where the other
29

structures and the building stand, which were not included in the
settlement.
(C)
Whether or not the respondent Court of Appeals erred in ordering the
removal of the structures and surrounding walls on the encroached area
and in withdrawing its earlier ruling in its August 28, 1992 decision for the
petitioner to pay for the value of the land occupied by the building, only
because the private respondent has manifested its choice to demolish it
despite the absence of compulsory sale where the builder fails to pay for the
land, and which choice private respondent deliberately deleted from its
September 1, 1980 answer to the supple-mental complaint in the Regional
Trial Court.
In its Memorandum, petitioner poses the following issues:
A
The time when to determine the good faith of the builder under Article 448
of the New Civil Code, is reckoned during the period when it was actually
being built; and in a case where no evidence was presentednor introduced
as to the good faith or bad faith of the builder at that time, as in this case,
he must bepresumed to be a builder in good faith, since bad
faith cannot be presumed.
[9]

B.
In a specific boundary overlap situation which involves a builder in good
faith, as in this case, it is now well settled that the lot owner, who builds on
the adjacent lot is not charged with constructive notice of the technical
metes and bounds contained in their torrens titles to determine the exact
and precise extent of his boundary perimeter.
[10]

C.
The respondent courts citation of the twin cases of Tuason & Co. v.
Lumanlan and Tuason & Co. v. Macalindong is not the judicial authority for
a boundary dispute situation between adjacent torrens titled lot owners, as
the facts of the present case do not fall within nor square with the involved
principle of a dissimilar case.
[11]

D.
30

Quite contrary to respondent Uys reasoning, petitioner Tecnogas continues
to be a builder in good faith, even if it subsequently built/repaired the
walls/other permanent structures thereon while the case a quo was pending
and even while respondent sent the petitioner many letters/filed cases
thereon.
[12]

D. (E.)
The amicable settlement between the parties should be interpreted as a
contract and enforced only in accordance with its explicit terms,
and not over and beyond that agreed upon; because the courts do nothave
the power to create a contract nor expand its scope.
[13]

E. (F.)
As a general rule, although the landowner has the option to choose
between: (1) buying the building built in good faith, or (2) selling the
portion of his land on which stands the building under Article 448 of the Civil
Code; the first option is not absolute, because an exception thereto, once it
would be impractical for the landowner to choose to exercise the first
alternative, i.e. buy that portion of the house standing on his land, for the
whole building might be rendered useless. The workable solution is for him
to select the second alternative, namely, to sell to the builder that part of his
land on which was constructed a portion of the house.
[14]

Private respondent, on the other hand, argues that the petition is
suffering from the following flaws:
[15]

1. It did not give the exact citations of cases decided by the Honorable
Supreme Court that allegedly contradicts the ruling of the Hon. Court
of Appeals based on the doctrine laid down in Tuason vs. Lumanlan
case citing also Tuason vs. Macalindong case (Supra).
2. Assuming that the doctrine in the alleged Co Tao vs. Chico case is
contradictory to the doctrine in Tuason vs. Lumanlan and Tuason vs.
Macalindong, the two cases being more current, the same should
prevail.
Further, private respondent contends that the following unmistakably point
to the bad faith of petitioner: (1) private respondents purchase of the two
lots, was ahead of the purchase by petitioner of the building and lot from
Pariz Industries; (2) the declaration of the General Manager of Tecnogas
that the sale between petitioner and Pariz Industries was not registered
because of some problems with China Banking Corporation; and (3) the
31

Deed of Sale in favor of petitioner was registered in its name only in the
month of May 1973.
[16]

The Courts Ruling
The petition should be granted.
Good Faith or Bad Faith
Respondent Court, citing the cases of J. M. Tuason & Co., Inc. vs. Vda.
de Lumanlan
[17]
and J. M. Tuason & Co., Inc. vs. Macalindong,
[18]
ruled that
petitioner cannot be considered in good faith because as a land owner, it is
presumed to know the metes and bounds of his own property, specially if
the same are reflected in a properly issued certificate of title. One who
erroneously builds on the adjoining lot should be considered a builder in
(b)ad (f)aith, there being presumptive knowledge of the Torrens title, the
area, and the extent of the boundaries.
[19]

We disagree with respondent Court. The two cases it relied upon do not
support its main pronouncement that a registered owner of land has
presumptive knowledge of the metes and bounds of its own land, and is
therefore in bad faith if he mistakenly builds on an adjoining land. Aside
from the fact that those cases had factual moorings radically different from
those obtaining here, there is nothing in those cases which would suggest,
however remotely, that bad faith is imputable to a registered owner of land
when a part of his building encroaches upon a neighbors land, simply
because he is supposedly presumed to know the boundaries of his land as
described in his certificate of title. No such doctrinal statement could have
been made in those cases because such issue was not before the Supreme
Court. Quite the contrary, we have rejected such a theory in Co Tao vs.
Chico,
[20]
where we held that unless one is versed in the science of
surveying, no one can determine the precise extent or location of his
property by merely examining his paper title.
There is no question that when petitioner purchased the land from Pariz
Industries, the buildings and other structures were already in existence. The
record is not clear as to who actually built those structures, but it may well
be assumed that petitioners predecessor-in-interest, Pariz Industries, did
so. Article 527 of the Civil Code presumes good faith, and since no proof
exists to show that the encroachment over a narrow, needle-shaped portion
of private respondents land was done in bad faith by the builder of the
encroaching structures, the latter should be presumed to have built them in
32

good faith.
[21]
It is presumed that possession continues to be enjoyed in the
same character in which it was acquired, until the contrary is
proved.
[22]
Good faith consists in the belief of the builder that the land he is
building on is his, and his ignorance of any defect or flaw in his
title.
[23]
Hence, such good faith, by law, passed on to Parizs successor,
petitioner in this case. Further, (w)here one derives title to property from
another, the act, declaration, or omission of the latter, while holding the
title, in relation to the property, is evidence against the former.
[24]
And
possession acquired in good faith does not lose this character except in case
and from the moment facts exist which show that the possessor is not
unaware that he possesses the thing improperly or wrongfully.
[25]
The good
faith ceases from the moment defects in the title are made known to the
possessor, by extraneous evidence or by suit for recovery of the property by
the true owner.
[26]

Recall that the encroachment in the present case was caused by a very
slight deviation of the erected wall (as fence) which was supposed to run in
a straight line from point 9 to point 1 of petitioners lot. It was an error
which, in the context of the attendant facts, was consistent with good
faith. Consequently, the builder, if sued by the aggrieved landowner for
recovery of possession, could have invoked the provisions of Art. 448 of the
Civil Code, which reads:
The owner of the land on which anything has been built, sown or planted in
good faith, shall have the right to appropriate as his own the works, sowing
or planting, after payment of the indemnity provided for in articles 546 and
548, or to oblige the one who built or planted to pay the price of the land,
and the one who sowed, the proper rent. However, the builder or planter
cannot be obliged to buy the land if its value is considerably more than that
of the building or trees. In such case, he shall pay reasonable rent, if the
owner of the land does not choose to appropriate the building or trees after
proper indemnity. The parties shall agree upon the terms of the lease and in
case of disagreement, the court shall fix the terms thereof.
The obvious benefit to the builder under this article is that, instead of being
outrightly ejected from the land, he can compel the landowner to make a
choice between the two options: (1) to appropriate the building by paying
the indemnity required by law, or (2) sell the land to the builder. The
landowner cannot refuse to exercise either option and compel instead the
owner of the building to remove it from the land.
[27]

The question, however, is whether the same benefit can be invoked by
petitioner who, as earlier stated, is not the builder of the offending
structures but possesses them as buyer.
We answer such question in the affirmative.
33

In the first place, there is no sufficient showing that petitioner was aware
of the encroachment at the time it acquired the property from Pariz
Industries. We agree with the trial court that various factors in evidence
adequately show petitioners lack of awareness thereof. In any case,
contrary proof has not overthrown the presumption of good faith under
Article 527 of the Civil Code, as already stated, taken together with the
disputable presumptions of the law on evidence. These presumptions state,
under Section 3 (a) of Rule 131 of the Rules of Court, that the person is
innocent of a crime or wrong; and under Section 3 (ff) of Rule 131, that the
law has been obeyed. In fact, private respondent Eduardo Uy himself was
unaware of such intrusion into his property until after 1971 when he hired a
surveyor, following his purchase of another adjoining lot, to survey all his
newly acquired lots. Upon being apprised of the encroachment, petitioner
immediately offered to buy the area occupied by its building -- a species of
conduct consistent with good faith.
In the second place, upon delivery of the property by Pariz Industries, as
seller, to the petitioner, as buyer, the latter acquired ownership of the
property. Consequently and as earlier discussed, petitioner is deemed to
have stepped into the shoes of the seller in regard to all rights of ownership
over the immovable sold, including the right to compel the private
respondent to exercise either of the two options provided under Article 448
of the Civil Code.
Estoppel
Respondent Court ruled that the amicable settlement entered into
between petitioner and private respondent estops the former from
questioning the private respondents right over the disputed property. It
held that by undertaking to demolish the fence under said settlement,
petitioner recognized private respondents right over the property, and
cannot later on compel private respondent to sell to it the land since
private respondent is under no obligation to sell.
[28]

We do not agree. Petitioner cannot be held in estoppel for entering into
the amicable settlement, the pertinent portions of which read:
[29]

That the parties hereto have agreed that the rear portion of the fence that
separates the property of the complainant and respondent shall be
demolished up to the back of the building housing the machineries which
demolision (sic) shall be undertaken by the complainant at anytime.
34

That the fence which serve(s) as a wall housing the electroplating
machineries shall not be demolished in the mean time which portion shall be
subject to negotiation by herein parties.
From the foregoing, it is clear that petitioner agreed only to the
demolition of a portion of the wall separating the adjoining properties of the
parties -- i.e. up to the back of the building housing the machineries. But
that portion of the fence which served as the wall housing the electroplating
machineries was not to be demolished. Rather, it was to be subject to
negotiation by herein parties. The settlement may have recognized the
ownership of private respondent but such admission cannot be equated with
bad faith. Petitioner was only trying to avoid a litigation, one reason for
entering into an amicable settlement.
As was ruled in Osmea vs. Commission on Audit,
[30]

A compromise is a bilateral act or transaction that is expressly
acknowledged as a juridical agreement by the Civil Code and is therein dealt
with in some detail. `A compromise, declares Article 2208 of said Code, `is
a contract whereby the parties, by making reciprocal concessions, avoid a
litigation or put an end to one already commenced.
xxx xxx xxx
The Civil Code not only defines and authorizes compromises, it in fact
encourages them in civil actions. Art. 2029 states that `The Court shall
endeavor to persuade the litigants in a civil case to agree upon some fair
compromise. x x x.
In the context of the established facts, we hold that petitioner did not
lose its rights under Article 448 of the Civil Code on the basis merely of the
fact that some years after acquiring the property in good faith, it learned
about -- and aptly recognized -- the right of private respondent to a portion
of the land occupied by its building. The supervening awareness of the
encroachment by petitioner does not militate against its right to claim the
status of a builder in good faith. In fact, a judicious reading of said Article
448 will readily show that the landowners exercise of his option can only
take place after the builder shall have come to know of the intrusion -- in
short, when both parties shall have become aware of it. Only then will the
occasion for exercising the option arise, for it is only then that both parties
will have been aware that a problem exists in regard to their property rights.
Options of Private Respondent
35

What then is the applicable provision in this case which private
respondent may invoke as his remedy: Article 448 or Article 450
[31]
of
the Civil Code?
In view of the good faith of both petitioner and private respondent, their
rights and obligations are to be governed by Art. 448. The essential fairness
of this codal provision has been pointed out by Mme. Justice Ameurfina
Melencio-Herrera, citing Manresa and applicable precedents, in the case of
Depra vs. Dumlao,
[32]
to wit:
Where the builder, planter or sower has acted in good faith, a conflict of
rights arises between the owners, and it becomes necessary to protect the
owner of the improvements without causing injustice to the owner of the
land. In view of the impracticality of creating a state of forced co-
ownership, the law has provided a just solution by giving the owner of the
land the option to acquire the improvements after payment of the proper
indemnity, or to oblige the builder or planter to pay for the land and the
sower to pay the proper rent. It is the owner of the land who is authorized to
exercise the option, because his right is older, and because, by the principle
of accession, he is entitled to the ownership of the accessory thing. (3
Manresa 213; Bernardo vs. Bataclan, 37 Off. Gaz. 1382; Co Tao vs. Chan
Chico, G. R. No. 49167, April 30, 1949; Article applied; see Cabral, et al. vs.
Ibanez [S.C.] 52 Off. Gaz. 217; Marfori vs. Velasco, [C.A.] 52 Off. Gaz.
2050).
The private respondents insistence on the removal of the encroaching
structures as the proper remedy, which respondent Court sustained in its
assailed Decisions, is thus legally flawed. This is not one of the remedies
bestowed upon him by law. It would be available only if and when he
chooses to compel the petitioner to buy the land at a reasonable price but
the latter fails to pay such price.
[33]
This has not taken place. Hence, his
options are limited to: (1) appropriating the encroaching portion of
petitioners building after payment of proper indemnity, or (2) obliging the
latter to buy the lot occupied by the structure. He cannot exercise a remedy
of his own liking.
Neither is petitioners prayer that private respondent be ordered to sell
the land
[34]
the proper remedy. While that was dubbed as the more
workable solution in Grana and Torralba vs. The Court of Appeals, et
al.,
[35]
it was not the relief granted in that case as the landowners were
directed to exercise within 30 days from this decision their option to either
buy the portion of the petitioners house on their land or sell to said
petitioners the portion of their land on which it stands.
[36]
Moreover, in
Grana and Torralba, the area involved was only 87 square meters while this
case involves 520 square meters
[37]
. In line with the case of Depra vs.
36

Dumlao,
[38]
this case will have to be remanded to the trial court for further
proceedings to fully implement the mandate of Art. 448. It is a rule of
procedure for the Supreme Court to strive to settle the entire controversy in
a single proceeding leaving no root or branch to bear the seeds of future
litigation.
[39]

Petitioner, however, must also pay the rent for the property occupied by
its building as prescribed by respondent Court from October 4, 1979, but
only up to the date private respondent serves notice of its option upon
petitioner and the trial court; that is, if such option is for private respondent
to appropriate the encroaching structure. In such event, petitioner would
have a right of retention which negates the obligation to pay rent.
[40]
The
rent should however continue if the option chosen is compulsory sale, but
only up to the actual transfer of ownership.
The award of attorneys fees by respondent Court against petitioner is
unwarranted since the action appears to have been filed in good
faith. Besides, there should be no penalty on the right to litigate.
[41]

WHEREFORE, premises considered, the petition is hereby GRANTED and
the assailed Decision and the Amended Decision are REVERSED and SET
ASIDE. In accordance with the case of Depra vs. Dumlao,
[42]
this case is
REMANDED to the Regional Trial Court of Pasay City, Branch 117, for further
proceedings consistent with Articles 448 and 546
[43]
of the Civil Code, as
follows:
The trial court shall determine:
a) the present fair price of private respondents 520 square-meter
area of land;
b) the increase in value (plus value) which the said area of 520
square meters may have acquired by reason of the existence of
the portion of the building on the area;
c) the fair market value of the encroaching portion of the building;
and
d) whether the value of said area of land is considerably more
than the fair market value of the portion of the building thereon.
2. After said amounts shall have been determined by competent evidence,
the regional trial court shall render judgment as follows:
a) The private respondent shall be granted a period of fifteen (15)
days within which to exercise his option under the law (Article 448,
37

Civil Code), whether to appropriate the portion of the building as
his own by paying to petitioner its fair market value, or to oblige
petitioner to pay the price of said area. The amounts to be
respectively paid by petitioner and private respondent, in
accordance with the option thus exercised by written notice of the
other party and to the court, shall be paid by the obligor within
fifteen (15) days from such notice of the option by tendering the
amount to the trial court in favor of the party entitled to receive it;
b) If private respondent exercises the option to oblige petitioner to
pay the price of the land but the latter rejects such purchase
because, as found by the trial court, the value of the land is
considerably more than that of the portion of the building,
petitioner shall give written notice of such rejection to private
respondent and to the trial court within fifteen (15) days from
notice of private respondents option to sell the land. In that
event, the parties shall be given a period of fifteen (15) days from
such notice of rejection within which to agree upon the terms of
the lease, and give the trial court formal written notice of the
agreement and its provisos. If no agreement is reached by the
parties, the trial court, within fifteen (15) days from and after the
termination of the said period fixed for negotiation, shall then fix
the terms of the lease provided that the monthly rental to be fixed
by the Court shall not be less than two thousand pesos (P2,000.00)
per month, payable within the first five (5) days of each calendar
month. The period for the forced lease shall not be more than two
(2) years, counted from the finality of the judgment, considering
the long period of time since 1970 that petitioner has occupied the
subject area. The rental thus fixed shall be increased by ten
percent (10%) for the second year of the forced lease. Petitioner
shall not make any further constructions or improvements on the
building. Upon expiration of the two-year period, or upon default
by petitioner in the payment of rentals for two (2) consecutive
months, private respondent shall be entitled to terminate the
forced lease, to recover his land, and to have the portion of the
building removed by petitioner or at latters expense. The rentals
herein provided shall be tendered by petitioner to the trial court for
payment to private respondent, and such tender shall constitute
evidence of whether or not compliance was made within the period
fixed by the said court.
c) In any event, petitioner shall pay private respondent an amount
computed at two thousand pesos (P2,000.00) per month as
reasonable compensation for the occupancy of private respondents
land for the period counted from October 4, 1979, up to the date
38

private respondent serves notice of its option to appropriate the
encroaching structures, otherwise up to the actual transfer of
ownership to petitioner or, in case a forced lease has to be
imposed, up to the commencement date of the forced lease
referred to in the preceding paragraph;
d) The periods to be fixed by the trial court in its decision shall be
non-extendible, and upon failure of the party obliged to tender to
the trial court the amount due to the obligee, the party entitled to
such payment shall be entitled to an order of execution for the
enforcement of payment of the amount due and for compliance
with such other acts as may be required by the prestation due the
obligee.
No costs.
SO ORDERED.
BARTOLOME ORTIZ, petitioner,
vs.
HON. UNION C. KAYANAN, in his capacity as Judge of the Court of
First Instance of Quezon, Branch IV; ELEUTERIO ZAMORA, QUIRINO
COMINTAN, VICENTE FERRO, AND GREGORIO
PAMISARAN,respondents.
Salonga, Ordo;ez, Yap, Sicat & Associates and Salvador, Ulgado & Carbon
for petitioner.
Jose A. Cusi for private respondents.

ANTONIO, J.:1wph1.t
Petition for certiorari and Prohibition with Preliminary Injunction to nullify the
Order of respondent Judge directing the execution of the final judgment in
Civil Case No. C-90, entitled "Bartolome Ortiz vs. Secretary of Agriculture
and Natural Resources, et al.," and the Writ of Execution issued to
implement said Order, allegedly for being inconsistent with the judgment
sought to be enforced.
Civil Case No. C-90 was filed by Bartolome Ortiz who sought the review
and/or annulment of the decision of the Secretary of Agriculture and Natural
Resources, giving preference to the sales applications of private respondents
Quirino Comintan and Eleuterio Zamora over Lot No. 5785, PLS-45, located
at Barrio Cabuluan, Calauag, Quezon.
39

I
The factual background of the case, as found by respondent Court, is as
follows:t.hqw
... The lot in controversy was formerly the subject of Homestead
Application No. 122417 of Martin Dolorico II, plaintiff's ward who
died on August 20, 1931; that since then it was plaintiff who
continued the cultivation and possession of the property, without
however filing any application to acquire title thereon; that in the
Homestead Application No. 122417, Martin Dolorico II named his
uncle, Martin Dolorico I as his heir and successor in interest, so
that in 1951 Martin Dolorico I executed an affidavit relinquishing
his rights over the property in favor of defendants Quirino
Comintan and Eleuterio Zamora, his grandson and son-in-law,
respectively, and requested the Director of Lands to cancel the
homestead application; that on the strength of the affidavit,
Homestead Application No. 122417 was cancelled and thereafter,
defendants Comintan and Zamora filed their respective sales
applications Nos. 8433 and 9258; that plaintiff filed his protest
on November 26, 1951 alleging that he should be given
preference to purchase the lot inasmuch as he is the actual
occupant and has been in continuous possession of the same
since 1931; and inspite of plaintiff's opposition, "Portion A" of
the property was sold at public auction wherein defendant
Comintan was the only bidder; that on June 8, 1957,
investigation was conducted on plaintiff's protest by Assistant
Public Lands Inspector Serapion Bauzon who submitted his
report to the Regional Land Officer, and who in turn rendered a
decision on April 9, 1958, dismissing plaintiff's claim and giving
due course to defendants' sales applications on the ground that
the relinquishment of the homestead rights of Martin Dolorico I
in favor of Comintan and Zamora is proper, the former having
been designated as successor in interest of the original
homestead applicant and that because plaintiff failed to
participate in the public auction, he is forever barred to claim the
property; that plaintiff filed a motion for reconsideration of this
decision which was denied by the Director of Lands in his order
dated June 10, 1959; that, finally, on appeal to the Secretary of
Agriculture and Natural Resources, the decision rendered by the
Regional Land Officer was affirmed in toto.
1

40

On March 22, 1966, respondent Court rendered judgment in the afore-
mentioned civil case, the dispositive portion of which reads as
follows:t.hqw
IN VIEW OF THE FOREGOING CONSIDERATIONS, judgment is
hereby rendered awarding Lot No. 5785-A of PLS-45, (Calauag
Public Land Subdivision) one-half portion of the property in
litigation located at Bo. Cabuluan, Calauag, Quezon, in favor of
defendant QUIRINO COMINTAN, being the successful bidder in
the public auction conducted by the bureau of Lands on April 18,
1955, and hereby giving due course to the Sales Application No.
9258 of defendant Eleuterio Zamora over the other half, Lot No.
5785-B of PLS-45, Calauag, without prejudice to the right of
plaintiff BARTOLOME ORTIZ to participate in the public bidding of
the same to be announced by the Bureau of Lands, Manila.
However, should plaintiff Bartolome Ortiz be not declared the
successful bidder thereof, defendants Quirino Comintan and
Eleuterio Zamora are ordered to reimburse jointly said plaintiff
the improvements he has introduced on the whole property in
the amount of THIRTEEN THOUSAND SIX HUNDRED THIRTY-
TWO (P13,632.00) PESOS, the latter having the right to retain
the property until after he has been fully paid therefor, without
interest since he enjoys the fruits of the property in
question, with prejudice and with costs again the plaintiff.
2

Plaintiff appealed the decision to the Court of Appeals.
Two (2) years after the rendition of the judgment by the court a quo, while
the case was pending appeal and upon petition of private respondents
Quirino Comintan and Eleuterio Zamora, respondent Court appointed
respondent Vicente Ferro, Clerk of Court, as Receiver to collect tolls on a
portion of the property used as a diversion road. On August 19, 1969, the
Court of Appeals issued a Resolution annulling the Order appointing the
Receiver. Subsequently, on February 19, 1970, the Appellate Court affirmed
the decision of the trial court. A petition for review on certiorari of the
decision of the Court of Appeals was denied by this Court on April 6, 1970.
At this point, private respondents filed a petition for appointment of a new
receiver with the court a quo. This petition was granted and the receiver was
reappointed. Petitioner sought the annulment of this Order with the Court of
Appeals, but said Court ruled that its decision had already become final and
that the records of the case were to be remanded to the trial court.
Not satisfied with such denial, petitioner filed a petitioner for certiorari,
prohibition and mandamus with preliminary injunction before this
41

Court,
3
praying for the annulment of the Order reappointing the Receiver.
On July 13, 1970, the petition was dismissed by this Court on the ground of
insufficient showing of grave abuse of discretion.
II
The judgment having become final and executory private respondents filed a
motion for the execution of the same, praying as follows:t.hqw
WHEREFORE, it is respectfully prayed of this Honorable Court to
order the issuance of a writ of execution in accordance with the
judgment of this Honorable Court, confirmed by the Court of
Appeals and the Supreme Court, commanding any lawful officer
to deliver to defendants Comintan and Zamora the land subject
of the decision in this case but allowing defendants to file a bond
in such amount as this Honorable Court may fix, in lieu of the
P13,632.00 required to be paid to plaintiff, conditioned that after
the accounting of the tools collected by plaintiff, there is still an
amount due and payable to said plaintiff, then if such amount is
not paid on demand, including the legal interests, said bond shall
be held answerable.
Ordering further the plaintiff to render an accounting of the tolls
he collected from March of 1967 to December 31, 1968 and from
September 1969 to March 31, 1970, and deliver said tolls
collected to the receiver and if judgment is already executed,
then to Quirino Comintan and Eleuterio Zamora; and,
Finally, to condemn plaintiff to pay moral damages for
withholding the tools which belong to your movant in an amount
this Court may deem just in the premises.
4

Acting upon the foregoing motion, respondent Judge issued an Order, dated
September 23, 1970, stating, among others, the following: t.hqw
The records further disclosed that from March 1967 to December
31, 1968, piaintiff Bartolome Ortiz collected tolls on a portion of
the propertv in question wherein he has not introduced anv
improvement particularlv on Lot No. 5785-A; PLS-45 awarded to
defendant Quirino Comintan, thru which vehicular traffic was
detoured or diverted, and again from September 1969 to March
31, 1970, the plaintiff resumed the collection of tools on the
same portion without rendering any accounting on said tolls to
the Receiver, who, was reappointed after submitting the required
42

bond and specifically authorized only to collect tolls leaving the
harvesting of the improvements to the plaintiff.
xxx xxx xxx
ln virtue of he findings of this Court as contained in the
dispositive portion of its decision, the defendants are jointly
obligated to pay the plaintiff in the amount of P13,632.00 as
reasonable value of the improvements he introduced on the
whole property in question, and that he has the right of retention
until fully paid. It can be gleaned from the motion of the
defendants that if plaintiff submits an accounting of the tolls he
collected during the periods above alluded to, their damages of
about P25,000.00 can more than offset their obligation of
P13,362.00 in favor of the plaintiff, thereafter the possession of
the land be delivered to the defendants since the decision of the
Supreme Court has already become final and executory, but in
the interregnum pending such accounting and recovery by the
Receiver of the tolls collected by the plaintiff, the defendants
pray that they allowed to put up a bond in lieu of the said
P13,632.00 to answer for damages of the former, if any.
On the other hand, plaintiff contends in his opposition, admitting
that the decision of the Supreme Court has become final and
executory; (1) the offer of a bond in lieu of payment of
P13,632.00 does not, and cannot, satisfy the condition imposed
in the decision of this Court which was affirmed in toto; (2) the
public sale of Portion "B" of the land has still to take place as
ordained before the decision could be executed; and, (3) that
whatever sums plaintiff may derive from the property cannot be
set off against what is due him for the improvements he made,
for which he has to be reimbursed as ordered.
xxx xxx xxx
Let it be known that plaintiff does not dispute his having
collected tolls during the periods from March 1967 to December
31, 1968 and from September 1969 to March 31, 1970. The
Supreme Court affirmed the decision of this Court its findings
that said tolls belong to the defendant, considering that the
same were collected on a portion of the land question where the
plaintiff did notintroduce any improvement. The reimbursement
to the plaintiff pertains only to the value of the improvements,
like coconut trees and other plants which he introduced on the
43

whole property. The tolls collected by the plaintiff on an
unimproved portion naturally belong to the defendants, following
the doctrine on accretion. Further, the reappointment of a
Receiver by this Court was upheld by the Supreme Court when it
denied the petition for certiorari filed by the plaintiff, bolstering
the legal claim of defendants over said tolls. Thus, the decision
of the Supreme Court rendered the decision of this Court
retroactive from March 22, 1966 although pending accounting of
the tolls collected by the plaintiff is justified and will not
prejudice anybody, but certainly would substantially satisfy the
conditions imposed in the decision. However, insofar as the one-
half portion "B" of the property, the decision may be executed
only after public sale by the Bureau of Lands shall be
accomplished.
WHEREFORE, finding the Motion for Execution filed by the
defendants to be meritorious, the same is granted; provided,
however, that they put up a bond equal the adjudicated amount
of P13,632.00 accruing in favor of the plaintiff, from a reputable
or recognized bonding or surety company, conditioned that after
an accounting of the tolls collected by the plaintiff should there
be found out any balance due and payable to him after
reckoning said obligation of P13,632.00 the bond shall be held
answerable therefor.
5

Accordingly, a Writ of Execution was issued after private respondent Quirino
Comintan had filed the required bond. The writ directed the Sheriff to
enforce the decision of the Court, and stated, part in, the
following:t.hqw
But should there be found any amount collectible after
accounting and deducting the amount of P3,632.00, you are
hereby ordered that of the goods and chattels of Bartolome Ortiz
of Bo. Kabuluan, Calauag, Quezon, be caused to be made any
excess in the above-metioned amount together with your lawful
fees and that you render same to defendant Quirino Comintan. If
sufficient personal property cannot be found thereof to satisfy
this execution and lawful fees thereon, then you are commanded
that of the lands and buildings of the said BARTOLOME ORTIZ
you make the said excess amount in the manner required by the
Rules of Court, and make return of your proceedings within this
Court within sixty (60) days from date of service.
44

You are also ordered to cause Bartolome Ortiz to vacate the
property within fifteen (15) days after service thereof the
defendant Quirino Comintan having filed the required bond in the
amount of THIRTEEN THOUSAND SIX HUNDRED THIRTY-TWO
(P13,632.00) PESOS.
6

On October 12, 1970, petitioner filed a Motion for Reconsideration of the
aforesaid Order and Writ of Execution, alleging:t.hqw
(a) That the respondent judge has no authority to place
respondents in possession of the property;
(b) That the Supreme Court has never affirmed any decision of
the trial court that tolls collected from the diversionary road on
the property, which is public land, belong to said respondents;
(c) That to assess petitioner a P25,000.00 liability for damages is
purely punitive imposition without factual or legal justification.
The foregoing Motion for Reconsideration was denied by respondent Judge
per Order dated November 18, 1970. Saod Order states, in part:t.hqw
It goes without saying that defendant Comintan is entitled to be
placed in possession of lot No. 5785-A of PLS-45 (Calauag Public
Land Subdivision) and enjoyment of the tolls from March, 1967
to March, 1968 and from September, 1969 to March 31, l970
which were received by plaintiff Bartolome Ortiz, collected from
the property by reason of the diversion road where vehicular
traffic was detoured. To defendant Comintan belongs the tolls
thus collected from a portion of the land awarded to him used as
a diversionary road by the doctrine of accretion and his right
over the same is ipso jure, there being no need of any action to
possess said addition. It is so because as consistently maintained
by the Supreme Court, an applicant who has complied with all
the terms and conditions which entitle him to a patent for a
particular tract of publlic land, acquires a vested right therein
and is to be regarded as equitable owner thereof so that even
without a patent, a perfected homestead or sales application is a
property right in the fullest sense, unaffectcd by the fact that the
paramount title is still in the Government and no subsequent law
can deprive him of that vested right The question of the actual
damages suffered by defendant Comintan by reason of the
unaccounted tolls received by plaintiff had already been fully
discussed in the order of September 23, 1970 and the Court is
45

honestly convinced and believes it to be proper and regular
under the circumstances.
Incidentally, the Court stands to correct itself when in the same
order, it directed the execution of he decision with respect to the
one-half portion "B" of the property only after the public sale by
the Bureau of Lands, the same being an oversight, it appearing
that the Sales Application of defendant Eleuterio Zamora had
already been recognized and full confirmed by the Supreme
Court.
In view thereof, finding the motion filed by plaintiff to be without
merit, the Court hereby denies the same and the order of
September 23, 1970 shall remain in full force subject to the
amendment that the execution of the decision with respect to
the one-half portion "B" shall not be conditioned to the public
sale by the Bureau of Lands.
SO ORDERED.
7

III
Petitioner thus filed the instant petition, contending that in having issued the
Order and Writ of Execution, respondent Court "acted without or in excess of
jurisdiction, and/or with grave abuse of discretion, because the said order
and writ in effect vary the terms of the judgment they purportedly seek to
enforce." He argued that since said judgment declared the petitioner a
possessor in good faith, he is entitled to the payment of the value of the
improvements introduced by him on the whole property, with right to retain
the land until he has been fully paid such value. He likewise averred that no
payment for improvements has been made and, instead, a bond therefor
had been filed by defendants (private respondents), which, according to
petitioner, is not the payment envisaged in the decision which would entitle
private respondents to the possession of the property. Furthermore, with
respect to portion "B", petitioner alleges that, under the decision, he has the
right to retain the same until after he has participated and lost in the public
bidding of the land to be conducted by the Bureau of Lands. It is claimed
that it is only in the event that he loses in the bidding that he can be legally
dispossessed thereof.
It is the position of petitioner that all the fruits of the property, including the
tolls collected by him from the passing vehicles, which according to the trial
court amounts to P25,000.00, belongs to petitioner and not to
defendant/private respondent Quirino Comintan, in accordance with the
46

decision itself, which decreed that the fruits of the property shall be in lieu of
interest on the amount to be paid to petitioner as reimbursement for
improvements. Any contrary opinion, in his view, would be tantamount to an
amendment of a decision which has long become final and executory and,
therefore, cannot be lawfully done.
Petitioner, therefore, prayed that: (1) a Writ of Preliminary Injunction be
issued enjoining the enforcement of the Orders of September 23, 1970 and
November 18, 1970, and the Writ of Execution issued thereto, or restoring
to petitioner the possession of the property if the private respondents had
been placed in possession thereof; (2) annulling said Orders as well as the
Writ of Execution, dissolving the receivership established over the property;
and (3) ordering private respondents to account to petitioner all the fruits
they may have gathered or collected from the property in question from the
time of petitioiier's illegal dispossession thereof.
On January 29, 1971, this Court issued the Writ of Preliminary Injunction.
On January 30, 1971, private respondents filed a Motion for Reconsideration
and/or Modification of the Order dated January 29, 1971. This was followed
by a Supplemental Motion for Reconsideration and Manifestation on February
3, 1971. In the latter motion, private respondents manifested that the
amount of P14,040.96, representing the amount decreed in the judgment as
reimbursement to petitioner for the improvements, plus interest for six
months, has already been deposited by them in court, "with the
understanding that said amount shall be turned over to the plaintiff after the
court a quo shall have determined the improvement on Lot 5785-A, and
subsequently the remaining balance of the deposit shall be delivered to the
petitioner (plaintiff therein) in the event he loses the bid for Lot 5785-B in
favor of private respondent Eleuterio Zamora."
8
The deposit is evidenced by
a certification made by the Clerk of the Court a quo.
9
Contending that said
deposit was a faithful compliance with the judgment of the trial court,
private respondent Quirino Comintan prayed for the dissolution of the Writ of
Injunction.
It appears that as a consequence of the deposit made by private
respondents, the Deputy, Sheriff of Calauag, Quezon ousted petitioner's
representative from the land in question and put private respondents in
possession thereof.
10

On March 10, 1971, petitioner filed a "Comment on Respondents' 'Motion for
Reconsideration' dated January 29, 1971' and 'Supplemental Motion for
Reconsideration and Manifestation,'" contending that the tender of deposit
mentioned in the Suplemental Motion was not really and officially made,
"inasmuch as the same is notsupported by any official receipt from the lower
47

court, or from its clerk or cashier, as required by law;" that said deposit does
not constitute sufficient compliance with the judgment sought to be
enforced, neither was it legally and validly made because the requisites for
consignation had not been complied with; that the tender of legal interest
for six months cannot substitute petitioner's enjoyment of the fruits of the
property as long as the judgment in Civil Case No. C-90 has not been
implemented in the manner decreed therein; that contrary to the allegations
of private respondents, the value of the improvements on the whole
property had been determined by the lower court, and the segregation of
the improvements for each lot should have been raised by them at the
opportune moment by asking for the modification of the decision before it
became final and executory; and that the tolls on the property constituted
"civil fruits" to which the petitioner is entitled under the terms of the
decision.
IV
The issue decisive of the controvery isafter the rendition by the trial court
of its judgment in Civil Case No. C-90 on March 22, 1966 confirming the
award of one-half of the property to Quirino Comintanwhether or not
petitioner is still entitled to retain for his own exclusive benefit all the fruits
of the property, such as the tolls collected by him from March 1967 to
December 1968, and September 1969 to March 31, 1970, amounting to
about P25,000.00. In other words, petitioner contends that so long as the
aforesaid amount of P13,632,00 decreed in the judgment representing the
expenses for clearing the land and the value of the coconuts and fruit trees
planted by him remains unpaid, he can appropriate for his exclusive benefit
all the fruits which he may derive from the property, without any obligation
to apply any portion thereof to the payment of the interest and the principal
of the debt.
We find this contention untenable.
There is no question that a possessor in good faith is entitled to the fruits
received before the possession is legally interrupted.
11
Possession in good
faith ceases or is legally interrupted from the moment defects in the title are
made known to the possessor, by extraneous evidence or by the filing of an
action in court by the true owner for the recovery of the property.
12
Hence,
all the fruits that the possessor may receive from the time he is summoned
in court, or when he answers the complaint, must be delivered and paid by
him to the owner or lawful possessor.
13

However, even after his good faith ceases, the possessor in fact can still
retain the property, pursuant to Article 546 of the New Civil Code, until he
48

has been fully reimbursed for all the necessary and useful expenses made by
him on the property. This right of retention has been considered as one of
the conglomerate of measures devised by the law for the protection of the
possessor in good faith. Its object is to guarantee the reimbursement of the
expenses, such as those for the preservation of the property,
14
or for the
enhancement of its utility or productivity.
15
It permits the actual possessor
to remain in possession while he has not been reimbursed by the person
who defeated him in the possession for those necessary expenses and useful
improvements made by him on the thing possessed. The principal
characteristic of the right of retention is its accessory character. It is
accessory to a principal obligation. Considering that the right of the
possessor to receive the fruits terminates when his good faith ceases, it is
necessary, in order that this right to retain may be useful, to concede to the
creditor the right to secure reimbursement from the fruits of the property by
utilizing its proceeds for the payment of the interest as well as the principal
of the debt while he remains in possession. This right of retention of the
property by the creditor, according to Scaevola, in the light of the provisions
of Article 502 of the Spanish Civil Code,
16
is considered not a coercive
measure to oblige the debtor to pay, depriving him temporarily of the
enjoyment of the fruits of his property, but as a means of obtainitig
compensation for the debt. The right of retention in this case is analogous to
a contract of antichresis and it cati be considered as a means of
extinguishing the obligation, inasmuch as the right to retain the thing lasts
only for the period necessary to enable the creditor to be reimbursed from
the fruits for the necessary and useful expenses.
17

According to Manresa, the right of retention is, therefore, analogous to that
of a pledge, if the property retained is a movable, and to that of antichresis,
if the property held is immovable.
18
This construction appears to be in
harmony with similar provisions of the civil law which employs the right of
retention as a means or device by which a creditor is able to obtain the
payment of a debt. Thus, under Article 1731 of the New Civil Code, any
person who has performed work upon a movable has a right to retain it by
way of pledge until he is paid. Similarly, under Article 1914 of the same
Code, the agent may retain in pledge the things which are the object of the
agency until the principal effects reimbursement of the funds advanced by
the former for the execution of the agency, or he is indemnified for all
damages which he may have suffered as a consequence of the execution of
the agency, provided he is free from fault. To the same effect, the
depositary, under Article 1994 of the same Code, may retain the thing in
pledge until the full payment of what may be due him by reason of the
deposit. The usufructuary, pursuant to Article 612 of the same Code, may
retain the property until he is reimbursed for the amount paid for taxes
levied on the capital (Article 597) and tor extraordinary repairs (Article 594).
49

In all of these cases, the right of retention is used as a means of
extinguishing the obligation. As amply observed by Manresa: "El derecho de
retencion, lo hemos dicho, es el derecho de prenda o el de anticresis
constituido por la ley con independencia de las partes."
19
In a pledge, if the
thing pledged earns or produces fruits, income, dividends or interests, the
creditor shall compensate what he receives with those which are owing
him.
20
In the same manner, in a contract of antichresis, the creditor
acquires the right to receive the fruits of an immovable of his debtor with
the obligation to apply them to payment of the interest, if owing, and
thereafter to the principal of his credit.
21
The debtor can not reacquire
enjoyment of the immovable until he has actually paid what he owes the
creditor.
22

Applying the afore-cited principles to the case at bar, petitioner cannot
appropriate for his own exclusive benefit the tolls which he collected from
the property retained by him. It was his duty under the law, after deducting
the necessary expenses for his administration, to apply such amount
collected to the payment of the interest, and the balance to the payment of
the obligation.
We hold, therefore, that the disputed tolls, after deducting petitioner's
expenses for administration, belong to Quirino Comintan, owner of the land
through which the toll road passed, further considering that the same was
on portions of the property on which petitioner had not introduced any
improvement. The trial court itself clarified this matter when it placed the toll
road under receivership. The omission of any mention of the tolls in the
decision itself may be attributed to the fact that the tolls appear to have
been collected after the rendition of the judgment of the trial court.
The records further reveal that earnest efforts have been made by private
respondents to have the judgment executed in the most practicable manner.
They deposited in court the amount of the judgment in the sum of
P13,632.00 in cash, subject only to the accounting of the tolls collected by
the petitioner so that whatever is due from him may be set off with the
amount of reimbursement. This is just and proper under the circumstances
and, under the law, compensation or set off may take place, either totally or
partially. Considering that petitioner is the creditor with respect to the
judgment obligation and the debtor with respect to the tolls collected,
Comintan being the owner thereof, the trial court's order for an accounting
and compensation is in accord with law.
23

With respect to the amount of reimbursement to be paid by Comintan, it
appears that the dispositive portion of the decision was lacking in specificity,
as it merely provided that Comintan and Zamora are jointly liable therefor.
50

When two persons are liable under a contract or under a judgment, and no
words appear in the contract or judgment to make each liable for the entire
obligation, the presumption is that their obligation is joint ormancomunada,
and each debtor is liable only for a proportionate part of the
obligation.
24
The judgment debt of P13,632.00 should, therefore, be pro-
rated in equal shares to Comintan and Zamora.
Regarding Lot 5785-B, it appears that no public sale has yet been conducted
by the Bureau of Lands and, therefore, petitioner is entitled to remain in
possession thereof. This is not disputed by respondent Eleuterio
Zamora.
25
After public sale is had and in the event that Ortiz is not declared
the successful bidder, then he should be reimbursed by respondent Zamora
in the corresponding amount for the improvements on Lot 5785-B.
WHEREFORE, in view hereof, the Order of respondent Court of November 18,
1970 is hereby modified to conform to the foregoing judgment. The Writ of
Preliminary Injunction, dated January 29, 1971, is hereby dissolved. Without
special pronouncement as to costs.
Barredo (Chairman), Concepcion, Jr. and Guerrero, JJ., concur.1wph1.t
Aquino, J., concurs in the result.
Santos and Abad Santos, JJ., are on leave.
Guerrero, J., was designated to sit in the Second Division
FEDERICO GEMINIANO, MARIA GEMINIANO, ERNESTO GEMINIANO,
ASUNCION GEMINIANO, LARRY GEMINIANO, and MARLYN
GEMINIANO,petitioners, vs. COURT OF APPEALS, DOMINADOR
NICOLAS, and MARY A. NICOLAS, respondents.
D E C I S I O N
DAVIDE, JR., J.:
This petition for review on certiorari has its origins in Civil Case No. 9214
of Branch 3 of the Municipal Trial Court in Cities (MTCC) in Dagupan City for
unlawful detainer and damages. The petitioners ask the Court to set aside
the decision of the Court of Appeals affirming the decision of Branch 40 of
the Regional Trial Court (RTC) of Dagupan City, which, in turn, reversed the
MTCC; ordered the petitioners to reimburse the private respondents the
value of the house in question and other improvements; and allowed the
latter to retain the premises until reimbursement was made.
51

It appears that Lot No. 3765-B-1 containing an area of 314 square
meters was originally owned by the petitioners' mother, Paulina Amado vda.
de Geminiano. On a 12-square-meter portion of that lot stood the
petitioners' unfinished bungalow, which the petitioners sold in November
1978 to the private respondents for the sum of P6,000.00, with an alleged
promise to sell to the latter that portion of the lot occupied by the
house. Subsequently, the petitioners' mother executed a contract of lease
over a 126 square-meter portion of the lot, including that portion on which
the house stood, in favor of the private respondents for P40.00 per month
for a period of seven years commencing on 15 November 1978.
[1]
The
private respondents then introduced additional improvements and registered
the house in their names. After the expiration of the lease contract in
November 1985, however, the petitioners' mother refused to accept the
monthly rentals.
It turned out that the lot in question was the subject of a suit, which
resulted in its acquisition by one Maria Lee in 1972. In 1982, Lee sold the
lot to Lily Salcedo, who in turn sold it in 1984 to the spouses Agustin and
Ester Dionisio.
On 14 February 1992, the Dionisio spouses executed a Deed of Quitclaim
over the said property in favor of the petitioners.
[2]
As such, the lot was
registered in the latter's names.
[3]

On 9 February 1993, the petitioners sent, via registered mail, a letter
addressed to private respondent Mary Nicolas demanding that she vacate
the premises and pay the rentals in arrears within twenty days from
notice.
[4]

Upon failure of the private respondents to heed the demand, the
petitioners filed with the MTCC of Dagupan City a complaint for unlawful
detainer and damages.
During the pre-trial conference, the parties agreed to confine the issues
to: (1) whether there was an implied renewal of the lease which expired in
November 1985; (2) whether the lessees were builders in good faith and
entitled to reimbursement of the value of the house and improvements; and
(3) the value of the house.
The parties then submitted their respective position papers and the case
was heard under the Rule on Summary Procedure.
On the first issue, the court held that since the petitioners' mother was
no longer the owner of the lot in question at the time the lease contract was
executed in 1978, in view of its acquisition by Maria Lee as early as 1972,
there was no lease to speak of, much less, a renewal thereof. And even if
the lease legally existed, its implied renewal was not for the period
52

stipulated in the original contract, but only on a month-to-month basis
pursuant to Article 1687 of the Civil Code. The refusal of the petitioners'
mother to accept the rentals starting January 1986 was then a clear
indication of her desire to terminate the monthly lease. As regards the
petitioners' alleged failed promise to sell to the private respondents the lot
occupied by the house, the court held that such should be litigated in a
proper case before the proper forum, not an ejectment case where the only
issue was physical possession of the property.
The court resolved the second issue in the negative, holding that Articles
448 and 546 of the Civil Code, which allow possessors in good faith to
recover the value of improvements and retain the premises until reimbursed,
did not apply to lessees like the private respondents, because the latter
knew that their occupation of the premises would continue only during the
life of the lease. Besides, the rights of the private respondents were
specifically governed by Article 1678, which allows reimbursement of up to
one-half of the value of the useful improvements, or removal of the
improvements should the lessor refuse to reimburse.
On the third issue, the court deemed as conclusive the private
respondents' allegation that the value of the house and improvements was
P180,000.00, there being no controverting evidence presented.
The trial court thus ordered the private respondents to vacate the
premises, pay the petitioners P40.00 a month as reasonable compensation
for their stay thereon from the filing of the complaint on 14 April 1993 until
they vacated, and to pay the sum of P1,000.00 as attorney's fees, plus
costs.
[5]

On appeal by the private respondents, the RTC of Dagupan City reversed
the trial court's decision and rendered a new judgment: (1) ordering the
petitioners to reimburse the private respondents for the value of the house
and improvements in the amount of P180,000.00 and to pay the latter
P10,000.00 as attorney's fees and P2,000.00 as litigation expenses; and (2)
allowing the private respondents to remain in possession of the premises
until they were fully reimbursed for the value of the house.
[6]
It ruled that
since the private respondents were assured by the petitioners that the lot
they leased would eventually be sold to them, they could be considered
builders in good faith, and as such, were entitled to reimbursement of the
value of the house and improvements with the right of retention until
reimbursement had been made.
On appeal, this time by the petitioners, the Court of Appeals affirmed the
decision of the RTC
[7]
and denied
[8]
the petitioners' motion for
reconsideration. Hence, the present petition.
53

The Court is confronted with the issue of which provision of law governs
the case at bench: Article 448 or Article 1678 of the Civil Code? The said
articles read as follows:
Art. 448. The owner of the land on which anything has been built, sown or
planted in good faith, shall have the right to appropriate as his own the
works, sowing or planting, after payment of the indemnity provided for in
articles 546 and 548, or to oblige the one who built or planted to pay the
price of the land, and the one who sowed, the proper rent. However, the
builder or planter cannot be obliged to buy the land if its value is
considerably more than that of the building or trees. In such case, he shall
pay reasonable rent, if the owner of the land does not choose to appropriate
the building or trees after proper indemnity. The parties shall agree upon
the terms of the lease and in case of disagreement, the court shall fix the
terms thereof.
xxx xxx xxx
Art. 1678. If the lessee makes, in good faith, useful improvements which
are suitable to the use for which the lease is intended, without altering the
form or substance of the property leased, the lessor upon the termination of
the lease shall pay the lessee one-half of the value of the improvements at
that time. Should the lessor refuse to reimburse said amount, the lessee
may remove the improvements, even though the principal thing may suffer
damage thereby. He shall not, however, cause any more impairment upon
the property leased than is necessary.
With regard to ornamental expenses, the lessee shall not be entitled to any
reimbursement, but he may remove the ornamental objects, provided no
damage is caused to the principal thing, and the lessor does not choose to
retain them by paying their value at the time the lease is extinguished.
The crux of the said issue then is whether the private respondents are
builders in good faith or mere lessees.
The private respondents claim they are builders in good faith, hence,
Article 448 of the Civil Code should apply. They rely on the lack of title of
the petitioners' mother at the time of the execution of the contract of lease,
as well as the alleged assurance made by the petitioners that the lot on
which the house stood would be sold to them.
It has been said that while the right to let property is an incident of title
and possession, a person may be a lessor and occupy the position of a
landlord to the tenant although he is not the owner of the premises
54

let.
[9]
After all, ownership of the property is not being transferred,
[10]
only
the temporary use and enjoyment thereof.
[11]

In this case, both parties admit that the land in question was originally
owned by the petitioners' mother. The land was allegedly acquired later by
one Maria Lee by virtue of an extrajudicial foreclosure of mortgage. Lee,
however, never sought a writ of possession in order that she gain possession
of the property in question.
[12]
The petitioners' mother therefore remained in
possession of the lot.
It is undisputed that the private respondents came into possession of a
126 square-meter portion of the said lot by virtue of a contract of lease
executed by the petitioners' mother in their favor. The juridical relation
between the petitioners' mother as lessor, and the private respondents as
lessees, is therefore well-established, and carries with it a recognition of the
lessor's title.
[13]
The private respondents, as lessees who had undisturbed
possession for the entire term under the lease, are then estopped to deny
their landlord's title, or to assert a better title not only in themselves, but
also in some third person while they remain in possession of the leased
premises and until they surrender possession to the landlord.
[14]
This
estoppel applies even though the lessor had no title at the time the relation
of lessor and lessee was created,
[15]
and may be asserted not only by the
original lessor, but also by those who succeed to his title.
[16]

Being mere lessees, the private respondents knew that their occupation
of the premises would continue only for the life of the lease. Plainly, they
cannot be considered as possessors nor builders in good faith.
[17]

In a plethora of cases,
[18]
this Court has held that Article 448 of the Civil
Code, in relation to Article 546 of the same Code, which allows full
reimbursement of useful improvements and retention of the premises until
reimbursement is made, applies only to a possessor in good faith,i.e., one
who builds on land with the belief that he is the owner thereof. It does not
apply where one's only interest is that of a lessee under a rental contract;
otherwise, it would always be in the power of the tenant to "improve" his
landlord out of his property.
Anent the alleged promise of the petitioners to sell the lot occupied by
the private respondents' house, the same was not substantiated by
convincing evidence. Neither the deed of sale over the house nor the
contract of lease contained an option in favor of the respondent spouses to
purchase the said lot. And even if the petitioners indeed promised to sell, it
would not make the private respondents possessors or builders in good faith
so as to be covered by the provisions of Article 448 of the Civil Code. The
latter cannot raise the mere expectancy of ownership of the aforementioned
lot because the alleged promise to sell was not fulfilled nor its existence
55

even proven. The first thing that the private respondents should have done
was to reduce the alleged promise into writing, because under Article 1403
of the Civil Code, an agreement for the sale of real property or an interest
therein is unenforceable, unless some note or memorandum thereof be
produced. Not having taken any steps in order that the alleged promise to
sell may be enforced, the private respondents cannot bank on that promise
and profess any claim nor color of title over the lot in question.
There is no need to apply by analogy the provisions of Article 448 on
indemnity as was done in Pecson vs. Court of Appeals,
[19]
because the
situation sought to be avoided and which would justify the application of that
provision, is not present in this case. Suffice it to say, "a state of forced co-
ownership" would not be created between the petitioners and the private
respondents. For, as correctly pointed out by the petitioners, the rights of
the private respondents as lessees are governed by Article 1678 of the Civil
Code which allows reimbursement to the extent of one-half of the value of
the useful improvements.
It must be stressed, however, that the right to indemnity under Article
1678 of the Civil Code arises only if the lessor opts to appropriate the
improvements. Since the petitioners refused to exercise that option,
[20]
the
private respondents cannot compel them to reimburse the one-half value of
the house and improvements. Neither can they retain the premises until
reimbursement is made. The private respondents' sole right then is to
remove the improvements without causing any more impairment upon the
property leased than is necessary.
[21]

WHEREFORE, judgment is hereby rendered GRANTING the instant
petition; REVERSING and SETTING ASIDE the decision of the Court of
Appeals of 27 January 1995 in CA-G.R. SP No. 34337; and REINSTATING the
decision of Branch 3 of the Municipal Trial Court in Cities of Dagupan City in
Civil Case No. 9214 entitled "Federico Geminiano, et al. vs. Dominador
Nicolas, et al."
Costs against the private respondents.
SO ORDERED.
PLEASANTVILLE DEVELOPMENT CORPORATION, petitioner,
vs. COURT OF APPEALS, WILSON KEE, C.T. TORRES
ENTERPRISES, INC. and ELDRED JARDINICO, respondents.
D E C I S I O N
PANGANIBAN, J.:
56

Is a lot buyer who constructs improvements on the wrong property
erroneously delivered by the owners agent, a builder in good faith? This is
the main issue resolved in this petition for review on certiorari to reverse the
Decision
[1]
of the Court of Appeals
[2]
in CA-G.R. SP No. 11040, promulgated
on August 20, 1987.
By resolution dated November 13, 1995, the First Division of this Court
resolved to transfer this case (along with several others) to the Third
Division. After due deliberation and consultation, the Court assigned the
writing of this Decision to the undersigned ponente.
The Facts
The facts, as found by respondent Court, are as follows:
Edith Robillo purchased from petitioner a parcel of land designated
as Lot 9, Phase II and located at Taculing Road, Pleasantville
Subdivision, Bacolod City. In 1975, respondent Eldred Jardinico bought the
rights to the lot from Robillo. At that time, Lot 9 was vacant.
Upon completing all payments, Jardinico secured from the Register of
Deeds of Bacolod City on December 19, 1978 Transfer Certificate of Title No.
106367 in his name. It was then that he discovered that improvements had
been introduced on Lot 9 by respondent Wilson Kee, who had taken
possession thereof.
It appears that on March 26, 1974, Kee bought on installment Lot 8 of
the same subdivision from C.T. Torres Enterprises, Inc. (CTTEI), the
exclusive real estate agent of petitioner. Under the Contract to Sell on
Installment, Kee could possess the lot even before the completion of all
installment payments. On January 20, 1975, Kee paid CTTEI the relocation
fee of P50.00 and another P50.00 on January 27, 1975, for the preparation
of the lot plan. These amounts were paid prior to Kees taking actual
possession of Lot 8. After the preparation of the lot plan and a copy thereof
given to Kee, CTTEI through its employee, Zenaida Octaviano, accompanied
Kees wife, Donabelle Kee, to inspect Lot 8. Unfortunately, the parcel of land
pointed by Octaviano wasLot 9. Thereafter, Kee proceeded to construct his
residence, a store, an auto repair shop and other improvements on the lot.
After discovering that Lot 9 was occupied by Kee, Jardinico confronted
him. The parties tried to reach an amicable settlement, but failed.
On January 30, 1981, Jardinicos lawyer wrote Kee, demanding that the
latter remove all improvements and vacate Lot 9. When Kee refused to
vacate Lot 9, Jardinico filed with the Municipal Trial Court in Cities, Branch 3,
Bacolod City (MTCC), a complaint for ejectment with damages against Kee.
57

Kee, in turn, filed a third-party complaint against petitioner and CTTEI.
The MTCC held that the erroneous delivery of Lot 9 to Kee was
attributable to CTTEI. It further ruled that petitioner and CTTEI could not
successfully invoke as a defense the failure of Kee to give notice of his
intention to begin construction required under paragraph 22 of the Contract
to Sell on Installment and his having built a sari-sari store without. the prior
approval of petitioner required under paragraph 26 of said contract, saying
that the purpose of these requirements was merely to regulate the type of
improvements to be constructed on the lot
[3]
.
However, the MTCC found that petitioner had already rescinded its
contract with Kee over Lot8 for the latters failure to pay the installments
due, and that Kee had not contested the rescission. The rescission was
effected in 1979, before the complaint was instituted. The MTCC concluded
that Kee no longer had any right over the lot subject of the contract between
him and petitioner. Consequently, Kee must pay reasonable rentals for the
use of Lot 9, and, furthermore, he cannot claim reimbursement for the
improvements he introduced on said lot.
The MTCC thus disposed:
IN VIEW OF ALL THE FOREGOING, judgment is hereby rendered as follows:
1. Defendant Wilson Kee is ordered to vacate tithe premises of Lot 9,
covered by TCT No. 106367 and to remove all structures and improvements
he introduced thereon;
2. Defendant Wilson Kee is ordered to pay to the plaintiff rentals at the rate
of P 15.00 a day computed from the time this suit was filed on March 12,
1981 until he actually vacates the premises. This amount shall bear
interests (sic) at the rate of 12 per cent (sic) per annum.
3. Third-Party Defendant CT. Torres Enterprises, Inc. and Pleasantville
Subdivision are ordered to pay the plaintiff jointly and severally the sum of
P3,000.00 as attorneys fees and P700.00 as cost and litigation expenses.
[4]

On appeal, the Regional Trial Court, Branch 48, Bacolod City (RTC) ruled
that petitioner and CTTEI were not at fault or were not negligent, there
being no preponderant evidence to show that they directly participated in
the delivery of Lot 9 to Kee.
[5]
It found Kee a builder in bad faith. It further
ruled that even assuming arguendo that Kee was acting in good faith, he
was, nonetheless, guilty of unlawfully usurping the possessory right of
Jardinico over Lot 9 from the time he was served with notice to vacate said
lot, and thus was liable for rental.
58

The RTC thus disposed:
WHEREFORE, the decision appealed from is affirmed with respect to the
order against the defendant to vacate the premises of Lot No. 9 covered by
Transfer Certificate of Title No. T-106367 of the land records of Bacolod City;
the removal of all structures and improvements introduced thereon at his
expense and the payment to plaintiff (sic) the sum of Fifteen (P 15.00)
Pesos a day as reasonable rental to be computed from January 30, 1981,
the date of the demand, and not from the date of the filing of the complaint,
until he had vacated (sic) the premises, with interest thereon at 12% per
annum. This Court further renders judgment against the defendant to pay
the plaintiff the sum of Three Thousand (P3,000.00) Pesos as attorneys
fees, plus costs of litigation.
The third-party complaint against Third-Party Defendants Pleasantville
Development Corporation and C.T. Torres Enterprises, Inc. is
dismissed. The order against Third-Party Defendants to pay attorneys fees
to plaintiff and costs of litigation is reversed.
[6]

Following the denial of his motion for reconsideration on October 20,
1986, Kee appealed directly to the Supreme Court, which referred the
matter to the Court of Appeals.
The appellate court ruled that Kee was a builder in good faith, as he was
unaware of the mix-up when he began construction of the improvements
on Lot 8. It further ruled that the erroneous delivery was due to the
negligence of CTTEI, and that such wrong delivery was likewise imputable to
its principal, petitioner herein. The appellate court also ruled that the award
of rentals was without basis.
Thus, the Court of Appeals disposed:
WHEREFORE, the petition is GRANTED, the appealed decision is REVERSED,
and judgment is rendered as follows:
1. Wilson Kee is declared a builder in good faith with respect to the
improvements he introduced on Lot 9, and is entitled to the rights
granted him under Articles 448, 546 and 548 of the New Civil
Code.
2. Third-party defendants C.T. Torres Enterprises, Inc. and
Pleasantville Development Corporation are solidarily liable under
the following circumstances:
a. If Eldred Jardinico decides to appropriate the improvements and,
thereafter, remove these structures, the third-party defendants
59

shall answer for all demolition expenses and the value of the
improvements thus destroyed or rendered useless;
b. If Jardinico prefers that Kee buy the land, the third-party
defendants shall answer for the amount representing the value
of Lot 9 that Kee should pay to Jardinico.
3. Third-party defendants C.T. Torres Enterprises, Inc. and
Pleasantville Development Corporation are ordered to pay in
solidum the amount of P3,000.00 to Jardinico as attorneys fees, as
well as litigation expenses.
4. The award of rentals to Jardinico is dispensed with.
Furthermore, the case is REMANDED to the court of origin for the
determination of the actual value of the improvements and the property
(Lot 9), as well as for further proceedings in conformity with Article 448 of
the New Civil Code.
[7]

Petitioner then filed the instant petition against Kee, Jardinico and CTTEI.
The Issues
The petition submitted the following grounds to justify a review of the
respondent Courts Decision, as follows:
1. The Court of Appeals has decided the case in a way probably not in
accord with law or the the (sic) applicable decisions of the Supreme Court on
third-party complaints, by ordering third-party defendants to pay the
demolition expenses and/or price of the land;
2. The Court of Appeals has so far departed from the accepted course of
judicial proceedings, by granting to private respondent-Kee the rights of a
builder in good faith in excess of what the law provides, thus enriching
private respondent Kee at the expense of the petitioner;
3. In the light of the subsequent events or circumstances which changed
the rights of the parties, it becomes imperative to set aside or at least
modify the judgment of the Court of Appeals to harmonize with justice and
the facts;
60

4. Private respondent-Kee in accordance with the findings of facts of the
lower court is clearly a builder in bad faith, having violated several
provisions of the contract to sell on installments;
5. The decision of the Court of Appeals, holding the principal, Pleasantville
Development Corporation (liable) for the acts made by the agent in excess
of its authority is clearly in violation of the provision of the law;
6. The award of attorneys fees is clearly without basis and is equivalent to
putting a premium in (sic) court litigation.
From these grounds, the issues could be re-stated as follows:
(1) Was Kee a builder in good faith?
(2) What is the liability, if any, of petitioner and its agent, C.T. Torres
Enterprises, Inc.? and
(3) Is the award of attorneys fees proper?
The First Issue: Good Faith
Petitioner contends that the Court of Appeals erred in reversing the RTCs
ruling that Kee was a builder in bad faith.
Petitioner fails to persuade this Court to abandon the findings and
conclusions of the Court of Appeals that Kee was a builder in good faith. We
agree with the following observation of the Court of Appeals:
The roots of the controversy can be traced directly to the errors committed
by CTTEI, when it pointed the wrong property to Wilson Kee and his wife. It
is highly improbable that a purchaser of a lot would knowingly and willingly
build his residence on a lot owned by another, deliberately exposing himself
and his family to the risk of being ejected from the land and losing all
improvements thereon, not to mention the social humiliation that would
follow.
Under the circumstances, Kee had acted in the manner of a prudent man in
ascertaining the identity of his property. Lot 8 is covered by Transfer
Certificate of Title No. T-69561, while Lot 9 is identified in Transfer
Certificate of Title No. T-106367. Hence, under the Torrens system of land
registration, Kee is presumed to have knowledge of the metes and bounds of
the property with which he is dealing. x x x
61

xxx xxx xxx
But as Kee is a layman not versed in the technical description of his
property, he had to find a way to ascertain that what was described in TCT
No. 69561 matched Lot 8. Thus, he went to the subdivision developers
agent and applied and paid for the relocation of the lot, as well as for the
production of a lot plan by CTTEIs geodetic engineer. Upon Kees receipt of
the map, his wife went to the subdivision site accompanied by CTTEIs
employee, Octaviano, who authoritatively declared that the land she was
pointing to was indeed Lot 8. Having full faith and confidence in the
reputation of CTTEI, and because of the companys positive identification of
the property, Kee saw no reason to suspect that there had been a
misdelivery. The steps Kee had taken to protect his interests were
reasonable. There was no need for him to have acted ex-abundantia
cautela, such as being present during the geodetic engineers relocation
survey or hiring an independent geodetic engineer to countercheck for
errors, for the final delivery of subdivision lots to their owners is part of the
regular course of everyday business of CTTEI. Because of CTTEIs blunder,
what Kee had hoped to forestall did in fact transpire. Kees efforts all went to
naught.
[8]

Good faith consists in the belief of the builder that the land he is building
on is his and his ignorance of any defect or flaw in his title.
[9]
And as good
faith is presumed, petitioner has the burden of proving bad faith on the part
of Kee.
[10]

At the time he built improvements on Lot 8, Kee believed that said lot
was what he bought from petitioner. He was not aware that the lot
delivered to him was not Lot 8. Thus, Kees good faith. Petitioner failed to
prove otherwise.
To demonstrate Kees bad faith, petitioner points to Kees violation of
paragraphs 22 and 26 of the Contract of Sale on Installment.
We disagree. Such violations have no bearing whatsoever on whether
Kee was a builder in good faith, that is, on his state of mind at the time he
built the improvements on Lot 9. These alleged violations may give rise to
petitioners cause of action against Kee under the said contract (contractual
breach), but may not be bases to negate the presumption that Kee was a
builder in good faith.
Petitioner also points out that, as found by the trial court, the Contract of
Sale on Installment covering Lot 8 between it and Kee was rescinded long
before the present action was instituted. This has no relevance on the
liability of petitioner, as such fact does not negate the negligence of its agent
62

in pointing out the wrong lot to Kee. Such circumstance is relevant only as
it gives Jardinico a cause of action for unlawful detainer against Kee.
Petitioner next contends that Kee cannot claim that another lot was
erroneously pointed out to him because the latter agreed to the following
provision in the Contract of Sale on Installment, to wit:
13. The Vendee hereby declares that prior to the execution of his contract
he/she has personally examined or inspected the property made subject-
matter hereof, as to its location, contours, as well as the natural condition of
the lots and from the date hereof whatever consequential change therein
made due to erosion, the said Vendee shall bear the expenses of the
necessary fillings, when the same is so desired by him/her.
[11]

The subject matter of this provision of the contract is the change of the
location, contour and condition of the lot due to erosion. It merely provides
that the vendee, having examined the property prior to the execution of the
contract, agrees to shoulder the expenses resulting from such change.
We do not agree with the interpretation of petitioner that Kee contracted
away his right to recover damages resulting from petitioners
negligence. Such waiver would be contrary to public policy and cannot be
allowed. Rights may be waived, unless the waiver is contrary to law, public
order, public policy, morals, or good customs, or prejudicial to a third person
with a right recognized by law.
[12]

The Second Issue: Petitioners Liability
Kee filed a third-party complaint against petitioner and CTTEI, which was
dismissed by the RTC after ruling that there was no evidence from which
fault or negligence on the part of petitioner and CTTEI can be inferred. The
Court of Appeals disagreed and found CTTEI negligent for the erroneous
delivery of the lot by Octaviano, its employee.
Petitioner does not dispute the fact that CTTEI was its agent. But it
contends that the erroneous delivery of Lot 9 to Kee was an act which was
clearly outside the scope of its authority, and consequently, CTTEI alone
should be liable. It asserts that while [CTTEI] was authorized to sell the lot
belonging to the herein petitioner, it was never authorized to deliver the
wrong lot to Kee.
[13]

Petitioners contention is without merit.
The rule is that the principal is responsible for the acts of the agent, done
within the scope of his authority, and should bear the damage caused to
63

third persons.
[14]
On the other hand, the agent who exceeds his authority is
personally liable for the damage.
[15]

CTTEI was acting within its authority as the sole real estate
representative of petitioner when it made the delivery to Kee. In acting
within its scope of authority, it was, however, negligent. It is this negligence
that is the basis of petitioners liability, as principal of CTTEI, per Articles
1909 and 1910 of the Civil Code.
Pending resolution of the case before the Court of Appeals, Jardinico and
Kee on July 24, 1987 entered into a deed of sale, wherein the former
sold Lot 9 to Kee. Jardinico and Kee did not inform the Court of Appeals of
such deal.
The deed of sale contained the following provision:
1. That Civil Case No. 3815 entitled Jardinico vs. Kee which is now
pending appeal with the Court of Appeals, regardless of the outcome of the
decision shall be mutually disregarded and shall not be pursued by the
parties herein and shall be considered dismissed and without effect
whatsoever;
[16]

Kee asserts though that the terms and conditions in said deed of sale
are strictly for the parties thereto and that (t)here is no waiver made by
either of the parties in said deed of whatever favorable judgment or award
the honorable respondent Court of Appeals may make in their favor against
herein petitioner Pleasantville Development Corporation and/or private
respondent C.T. Torres Enterprises, Inc.
[17]

Obviously, the deed of sale can have no effect on the liability of
petitioner. As we have earlier stated, petitioners liability is grounded on the
negligence of its agent. On the other hand, what the deed of sale regulates
are the reciprocal rights of Kee and Jardinico; it stressed that they had
reached an agreement independent of the outcome of the case.
Petitioner further assails the following holding of the Court of Appeals:
2. Third-party defendants C.T. Torres Enterprises, Inc. and Pleasantville
Development Corporation are solidarily liable under the following
circumstances:
a. If Eldred Jardinico decides to appropriate the improvements and,
thereafter, remove these structures, the third-party defendants
shall answer for all demolition expenses and the value of the
improvements thus destroyed or rendered useless;
64

b. If Jardinico prefers that Kee buy the land, the third-party
defendants shall answer for the amount representing the value
of Lot 9 that Kee should pay to Jardinico.
[18]

Petitioner contends that if the above holding would be carried out, Kee
would be unjustly enriched at its expense. In other words, Kee would be -
able to own the lot, as buyer, without having to pay anything on it, because
the aforequoted portion of respondent Courts Decision would require
petitioner and CTTEI jointly and solidarily to answer or reimburse Kee
there for.
We agree with petitioner.
Petitioners liability lies in the negligence of its agent CTTEI. For such
negligence, the petitioner should be held liable for damages. Now, the
extent and/or amount of damages to be awarded is a factual issue which
should be determined after evidence is adduced. However, there is no
showing that such evidence was actually presented in the trial court; hence
no damages could now be awarded.
The rights of Kee and Jardinico vis-a-vis each other, as builder in good
faith and owner in good faith, respectively, are regulated by law (i.e., Arts.
448, 546 and 548 of the Civil Code). It was error for the Court of Appeals to
make a slight modification in the application of such law, on the ground of
equity. At any rate, as it stands now, Kee and Jardinico have amicably
settled through their deed of sale their rights and obligations with regards
to Lot 9. Thus, we delete items 2 (a) and (b) of the dispositive portion of
the Court of Appeals Decision [as reproduced above] holding petitioner and
CTTEI solidarily liable.
The Third Issue: Attorneys Fees
The MTCC awarded Jardinico attorneys fees and costs in the amount of
P3,000.00 and P700.00, respectively, as prayed for in his complaint. The
RTC deleted the award, consistent with its ruling that petitioner was without
fault or negligence. The Court of Appeals, however, reinstated the award of
attorneys fees after ruling that petitioner was liable for its agents
negligence.
The award of attorneys fees lies within the discretion of the court and
depends upon the circumstances of each case.
[19]
We shall not interfere with
the discretion of the Court of Appeals. Jardinico was compelled to litigate for
the protection of his interests and for the recovery of damages sustained as
a result of the negligence of petitioners agent.
[20]

65

In sum, we rule that Kee is a builder in good faith. The disposition of the
Court of Appeals that Kee is entitled to the rights granted him under
Articles 448, 546 and 548 of the New Civil Code is deleted, in view of the
deed of sale entered into by Kee and Jardinico, which deed now governs the
rights of Jardinico and Kee as to each other. There is also no further need,
as ruled by the appellate Court, to remand the case to the court of origin
for determination of the actual value of the improvements and the property
(Lot 9), as well as for further proceedings in conformity with Article 448 of
the New Civil Code.
WHEREFORE, the petition is partially GRANTED. The Decision of the
Court of Appeals is hereby MODIFIED as follows:
(1) Wilson Kee is declared a builder in good faith;
(2) Petitioner Pleasantville Development Corporation and
respondent C.T. Tones Enterprises, Inc. are declared solidarily
liable for damages due to negligence; however, since the amount
and/or extent of such damages was not proven during the trial,
the same cannot now be quantified and awarded;
(3) Petitioner Pleasantville Develpment Corporation and
respondent C.T. Torres Enterprises, Inc. are ordered to pay
in solidum the amount of P3,000.00 to Jardinico as attorneys
fees, as well as litigation expenses; and
(4) The award of rentals to Jardinico is dispensed with.
SO ORDERED.
SPOUSES JUAN NUGUID AND ERLINDA T. NUGUID, petitioners,
vs. HON. COURT OF APPEALS AND PEDRO P.
PECSON, respondents.
D E C I S I O N
QUISUMBING, J.:
This is a petition for review on certiorari of the Decision
[1]
dated May 21,
2001, of the Court of Appeals in CA-G.R. CV No. 64295, which modified the
Order dated July 31, 1998 of the Regional Trial Court (RTC) of Quezon City,
Branch 101 in Civil Case No. Q-41470. The trial court ordered the
defendants, among them petitioner herein Juan Nuguid, to pay respondent
herein Pedro P. Pecson, the sum of P1,344,000 as reimbursement of
unrealized income for the period beginning November 22, 1993 to December
1997. The appellate court, however, reduced the trial courts award in favor
of Pecson from the said P1,344,000 to P280,000. Equally assailed by the
66

petitioners is the appellate courts Resolution
[2]
dated January 10, 2002,
denying the motion for reconsideration.
It may be recalled that relatedly in our Decision dated May 26, 1995, in
G.R. No. 115814, entitled Pecson v. Court of Appeals, we set aside the
decision of the Court of Appeals in CA-G.R. SP No. 32679 and the Order
dated November 15, 1993, of the RTC of Quezon City, Branch 101 and
remanded the case to the trial court for the determination of the current
market value of the four-door two-storey apartment building on the 256-
square meter commercial lot.
The antecedent facts in this case are as follows:
Pedro P. Pecson owned a commercial lot located at 27 Kamias
Road, Quezon City, on which he built a four-door two-storey apartment
building. For failure to pay realty taxes, the lot was sold at public auction by
the City Treasurer of Quezon City to Mamerto Nepomuceno, who in turn sold
it for P103,000 to the spouses Juan and Erlinda Nuguid.
Pecson challenged the validity of the auction sale before the RTC of
Quezon City in Civil Case No. Q-41470. In its Decision,
[3]
dated February 8,
1989, the RTC upheld the spouses title but declared that the four-door two-
storey apartment building was not included in the auction sale.
[4]
This was
affirmed in toto by the Court of Appeals and thereafter by this Court, in its
Decision
[5]
dated May 25, 1993, in G.R. No. 105360 entitled Pecson v. Court
of Appeals.
On June 23, 1993, by virtue of the Entry of Judgment of the aforesaid
decision in G.R. No. 105360, the Nuguids became the uncontested owners of
the 256-square meter commercial lot.
As a result, the Nuguid spouses moved for delivery of possession of the
lot and the apartment building.
In its Order
[6]
of November 15, 1993, the trial court, relying upon Article
546
[7]
of the Civil Code, ruled that the Spouses Nuguid were to reimburse
Pecson for his construction cost ofP53,000, following which, the
spouses Nuguid were entitled to immediate issuance of a writ of possession
over the lot and improvements. In the same order the RTC also directed
Pecson to pay the same amount of monthly rentals to the Nuguids as paid
by the tenants occupying the apartment units or P21,000 per month from
June 23, 1993, and allowed the offset of the amount of P53,000 due from
the Nuguids against the amount of rents collected by Pecson from June 23,
1993 to September 23, 1993 from the tenants of the apartment.
[8]

Pecson duly moved for reconsideration, but on November 8, 1993,
the RTC issued a Writ of Possession,
[9]
directing the deputy sheriff to put the
67

spouses Nuguid in possession of the subject property with all the
improvements thereon and to eject all the occupants therein.
Aggrieved, Pecson then filed a special civil action for certiorari and
prohibition docketed as CA-G.R. SP No. 32679 with the Court of Appeals.
In its decision of June 7, 1994, the appellate court, relying upon Article
448
[10]
of the Civil Code, affirmed the order of payment of construction costs
but rendered the issue of possession moot on appeal, thus:
WHEREFORE, while it appears that private respondents [spouses Nuguid]
have not yet indemnified petitioner [Pecson] with the cost of the
improvements, since Annex I shows that the Deputy Sheriff has enforced the
Writ of Possession and the premises have been turned over to the
possession of private respondents, the quest of petitioner that he be
restored in possession of the premises is rendered moot and academic,
although it is but fair and just that private respondents pay petitioner the
construction cost of P53,000.00; and that petitioner be ordered to account
for any and all fruits of the improvements received by him starting on June
23, 1993, with the amount of P53,000.00 to be offset therefrom.
IT IS SO ORDERED.
[11]
[Underscoring supplied.]
Frustrated by this turn of events, Pecson filed a petition for review
docketed as G.R. No. 115814 before this Court.
On May 26, 1995, the Court handed down the decision in G.R. No
115814, to wit:
WHEREFORE, the decision of the Court of Appeals in CA-G.R. SP No. 32679
and the Order of 15 November 1993 of the Regional Trial Court, Branch
101, Quezon City in Civil Case No. Q-41470 are hereby SET ASIDE.
The case is hereby remanded to the trial court for it to determine the current
market value of the apartment building on the lot. For this purpose, the
parties shall be allowed to adduce evidence on the current market value of
the apartment building. The value so determined shall be forthwith paid by
the private respondents [Spouses Juan and Erlinda Nuguid] to the petitioner
[Pedro Pecson] otherwise the petitioner shall be restored to the possession
of the apartment building until payment of the required indemnity.
No costs.
SO ORDERED.
[12]
[Emphasis supplied.]
68

In so ruling, this Court pointed out that: (1) Article 448 of the Civil Code
is not apposite to the case at bar where the owner of the land is the builder,
sower, or planter who then later lost ownership of the land by sale, but may,
however, be applied by analogy; (2) the current market value of the
improvements should be made as the basis of reimbursement; (3) Pecson
was entitled to retain ownership of the building and, necessarily, the income
therefrom; (4) the Court of Appeals erred not only in upholding the trial
courts determination of the indemnity, but also in ordering Pecson to
account for the rentals of the apartment building from June 23, 1993 to
September 23, 1993.
On the basis of this Courts decision in G.R. No. 115814, Pecson filed a
Motion to Restore Possession and a Motion to Render Accounting, praying
respectively for restoration of his possession over the subject 256-square
meter commercial lot and for the spouses Nuguid to be directed to render an
accounting under oath, of the income derived from the subject four-door
apartment from November 22, 1993 until possession of the same was
restored to him.
In an Order
[13]
dated January 26, 1996, the RTC denied the Motion to
Restore Possession to the plaintiff averring that the current market value of
the building should first be determined. Pending the said determination, the
resolution of the Motion for Accounting was likewise held in abeyance.
With the submission of the parties assessment and the reports of the
subject realty, and the reports of the Quezon City Assessor, as well as the
members of the duly constituted assessment committee, the trial court
issued the following Order
[14]
dated October 7, 1997, to wit:
On November 21, 1996, the parties manifested that they have arrived at a
compromise agreement that the value of the said improvement/building
is P400,000.00 The Court notes that the plaintiff has already
receivedP300,000.00. However, when defendant was ready to pay the
balance of P100,000.00, the plaintiff now insists that there should be a
rental to be paid by defendants. Whether or not this should be paid by
defendants, incident is hereby scheduled for hearing on November 12,
1997 at 8:30 a.m.
Meantime, defendants are directed to pay plaintiff the balance
of P100,000.00.
SO ORDERED.
[15]

On December 1997, after paying the said P100,000 balance to Pedro
Pecson the spousesNuguid prayed for the closure and termination of the
case, as well as the cancellation of the notice of lis pendens on the title of
69

the property on the ground that Pedro Pecsons claim for rentals was devoid
of factual and legal bases.
[16]

After conducting a hearing, the lower court issued an Order dated July
31, 1998, directing the spouses to pay the sum of P1,344,000 as
reimbursement of the unrealized income of Pecson for the period
beginning November 22, 1993 up to December 1997. The sum was based
on the computation of P28,000/month rentals of the four-door apartment,
thus:
The Court finds plaintiffs motion valid and meritorious. The decision of the
Supreme Court in the aforesaid case [Pecson vs. Court of Appeals, 244 SCRA
407] which set aside the Order of this Court of November 15, 1993 has in
effect upheld plaintiffs right of possession of the building for as long as he is
not fully paid the value thereof. It follows, as declared by the Supreme
Court in said decision that the plaintiff is entitled to the income derived
therefrom, thus
. . .
Records show that the plaintiff was dispossessed of the premises
on November 22, 1993 and that he was fully paid the value of his building in
December 1997. Therefore, he is entitled to the income thereof beginning
on November 22, 1993, the time he was dispossessed, up to the time of said
full payment, in December 1997, or a total of 48 months.
The only question left is the determination of income of the four units of
apartments per month. But as correctly pointed out by plaintiff, the
defendants have themselves submitted their affidavits attesting that the
income derived from three of the four units of the apartment building is
P21,000.00 or P7,000.00 each per month, or P28,000.00 per month for the
whole four units. Hence, at P28,000.00 per month, multiplied by 48
months, plaintiff is entitled to be paid by defendants the amount of
P1,344,000.00.
[17]

The Nuguid spouses filed a motion for reconsideration but this was
denied for lack of merit.
[18]

The Nuguid couple then appealed the trial courts ruling to the Court of
Appeals, their action docketed as CA-G.R. CV No. 64295.
In the Court of Appeals, the order appealed from in CA-G.R. CV No.
64295, was modified. The CA reduced the rentals from P1,344,000
to P280,000 in favor of the appellee.
[19]
The said amount represents accrued
rentals from the determination of the current market value on January 31,
1997
[20]
until its full payment on December 12, 1997.
70

Hence, petitioners state the sole assignment of error now before us as
follows:
THE COURT OF APPEALS ERRED IN HOLDING PETITIONERS LIABLE TO PAY
RENT OVER AND ABOVE THE CURRENT MARKET VALUE OF THE
IMPROVEMENT WHEN SUCH WAS NOT PROVIDED FOR IN THE DISPOSITIVE
PORTION OF THE SUPREME COURTS RULING IN G.R. No. 115814.
Petitioners call our attention to the fact that after reaching an agreed
price of P400,000 for the improvements, they only made a partial payment
of P300,000. Thus, they contend that their failure to pay the full price for
the improvements will, at most, entitle respondent to be restored to
possession, but not to collect any rentals. Petitioners insist that this is the
proper interpretation of the dispositive portion of the decision in G.R. No.
115814, which states in part that [t]he value so determined shall be
forthwith paid by the private respondents [Spouses Juan and Erlinda Nuguid]
to the petitioner [Pedro Pecson] otherwise the petitioner shall be restored
to the possession of the apartment building until payment of the required
indemnity.
[21]

Now herein respondent, Pecson, disagrees with herein petitioners
contention. He argues that petitioners are wrong in claiming that inasmuch
as his claim for rentals was not determined in the dispositive portion of the
decision in G.R. No. 115814, it could not be the subject of execution. He
points out that in moving for an accounting, all he asked was that the value
of the fruits of the property during the period he was dispossessed be
accounted for, since this Court explicitly recognized in G.R. No. 115814, he
was entitled to the property. He points out that this Court ruled that [t]he
petitioner [Pecson] not having been so paid, he was entitled to retain
ownership of the building and, necessarily, the income therefrom.
[22]
In
other words, says respondent, accounting was necessary. For accordingly,
he was entitled to rental income from the property. This should be given
effect. The Court could have very well specifically included rent (as fruit or
income of the property), but could not have done so at the time the Court
pronounced judgment because its value had yet to be determined, according
to him. Additionally, he faults the appellate court for modifying the order of
the RTC, thus defeating his right as a builder in good faith entitled to rental
from the period of his dispossession to full payment of the price of his
improvements, which spans from November 22, 1993 to December 1997, or
a period of more than four years.
It is not disputed that the construction of the four-door two-storey
apartment, subject of this dispute, was undertaken at the time when Pecson
was still the owner of the lot. When the Nuguids became the uncontested
owner of the lot on June 23, 1993, by virtue of entry of judgment of the
71

Courts decision, dated May 25, 1993, in G.R. No. 105360, the apartment
building was already in existence and occupied by tenants. In its
decision dated May 26, 1995 in G.R. No. 115814, the Court declared the
rights and obligations of the litigants in accordance with Articles 448 and 546
of the Civil Code. These provisions of the Code are directly applicable to the
instant case.
Under Article 448, the landowner is given the option, either to
appropriate the improvement as his own upon payment of the proper
amount of indemnity or to sell the land to the possessor in good
faith. Relatedly, Article 546 provides that a builder in good faith is entitled
to full reimbursement for all the necessary and useful expenses incurred; it
also gives him right of retention until full reimbursement is made.
While the law aims to concentrate in one person the ownership of the
land and the improvements thereon in view of the impracticability of
creating a state of forced co-ownership,
[23]
it guards against unjust
enrichment insofar as the good-faith builders improvements are
concerned. The right of retention is considered as one of the measures
devised by the law for the protection of builders in good faith. Its object is
to guarantee full and prompt reimbursement as it permits the actual
possessor to remain in possession while he has not been reimbursed (by the
person who defeated him in the case for possession of the property) for
those necessary expenses and useful improvements made by him on the
thing possessed.
[24]
Accordingly, a builder in good faith cannot be compelled
to pay rentals during the period of retention
[25]
nor be disturbed in his
possession by ordering him to vacate. In addition, as in this case, the owner
of the land is prohibited from offsetting or compensating the necessary and
useful expenses with the fruits received by the builder-possessor in good
faith. Otherwise, the security provided by law would be impaired. This is so
because the right to the expenses and the right to the fruits both pertain to
the possessor, making compensation juridically impossible; and one cannot
be used to reduce the other.
[26]

As we earlier held, since petitioners opted to appropriate the
improvement for themselves as early as June 1993, when they applied for a
writ of execution despite knowledge that the auction sale did not include the
apartment building, they could not benefit from the lots improvement, until
they reimbursed the improver in full, based on the current market value of
the property.
Despite the Courts recognition of Pecsons right of ownership over the
apartment building, the petitioners still insisted on dispossessing Pecson by
filing for a Writ of Possession to cover both the lot and the building. Clearly,
this resulted in a violation of respondents right of retention. Worse,
petitioners took advantage of the situation to benefit from the highly valued,
72

income-yielding, four-unit apartment building by collecting rentals thereon,
before they paid for the cost of the apartment building. It was only four
years later that they finally paid its full value to the respondent.
Petitioners interpretation of our holding in G.R. No. 115814 has neither
factual nor legal basis. The decision of May 26, 1995, should be construed
in connection with the legal principles which form the basis of the decision,
guided by the precept that judgments are to have a reasonable intendment
to do justice and avoid wrong.
[27]

The text of the decision in G.R. No. 115814 expressly exempted Pecson
from liability to pay rentals, for we found that the Court of Appeals erred not
only in upholding the trial courts determination of the indemnity, but also in
ordering him to account for the rentals of the apartment building from June
23, 1993 to September 23, 1993, the period from entry of judgment
untilPecsons dispossession. As pointed out by Pecson,
the dispositive portion of our decision in G.R. No. 115814 need not
specifically include the income derived from the improvement in order to
entitle him, as a builder in good faith, to such income. The right of
retention, which entitles the builder in good faith to the possession as well as
the income derived therefrom, is already provided for under Article 546 of
the Civil Code.
Given the circumstances of the instant case where the builder in good
faith has been clearly denied his right of retention for almost half a decade,
we find that the increased award of rentals by the RTC was reasonable and
equitable. The petitioners had reaped all the benefits from the improvement
introduced by the respondent during said period, without paying any amount
to the latter as reimbursement for his construction costs and
expenses. They should account and pay for such benefits.
We need not belabor now the appellate courts recognition of herein
respondents entitlement to rentals from the date of the determination of the
current market value until its full payment. Respondent is clearly entitled to
payment by virtue of his right of retention over the said improvement.
WHEREFORE, the instant petition is DENIED for lack of merit. The
Decision dated May 21, 2001 of the Court of Appeals in CA-G.R. CV No.
64295 is SET ASIDE and the Order dated July 31, 1998, of the Regional Trial
Court, Branch 101, Quezon City, in Civil Case No. Q-41470 ordering the
herein petitioners, Spouses Juan and Erlinda Nuguid, to account for the
rental income of the four-door two-storey apartment building from
November 1993 until December 1997, in the amount of P1,344,000,
computed on the basis of Twenty-eight Thousand (P28,000.00) pesos
monthly, for a period of 48 months, is hereby REINSTATED. Until fully paid,
said amount of rentals should bear the legal rate of interest set at six
73

percent (6%) per annum computed from the date of RTC judgment. If any
portion thereof shall thereafter remain unpaid, despite notice of finality of
this Courts judgment, said remaining unpaid amount shall bear the rate of
interest set at twelve percent (12%) per annum computed from the date of
said notice. Costs against petitioners.
SO ORDERED.
IGNACIO GRANDE, ET AL., petitioners,
vs.
HON. COURT OF APPEALS, DOMINGO CALALUNG, and ESTEBAN
CALALUNG, respondents.
Bartolome Guirao and Antonio M. Orara for petitioners.
Gonzales and Fernandez for respondents.
BARRERA, J.:
This is an appeal taken by petitioners Ignacio, Eulogia, Alfonso, Eulalia, and
Sofia Grande, from the decision of the Court of Appeals (CA-G.R. No. 25169-
R) reversing that of the Court of First Instance of Isabela (Civil Case No.
1171), and dismissing petitioners' action against respondents Domingo and
Esteban Calalung, to quiet title to and recover possession of a parcel of land
allegedly occupied by the latter without petitioners' consent.
The facts of the case, which are undisputed, briefly are: Petitioners are the
owners of a parcel of land, with an area of 3.5032 hectares, located at barrio
Ragan, municipality of Magsaysay (formerly Tumauini), province of Isabela,
by inheritance from their deceased mother Patricia Angui (who inherited it
from her parents Isidro Angui and Ana Lopez, in whose name said land
appears registered, as shown by Original Certificate of Title No. 2982, issued
on June 9, 1934). Said property is identified as Lot No. 1, Plan PSU-83342.
When it was surveyed for purposes of registration sometime in 1930, its
northeastern boundary was the Cagayan River (the same boundary stated in
the title). Since then, and for many years thereafter, a gradual accretion on
the northeastern side took place, by action of the current of the Cagayan
River, so much so, that by 1958, the bank thereof had receded to a distance
of about 105 meters from its original site, and an alluvial deposit of 19,964
square meters (1.9964 hectares), more or less, had been added to the
registered area (Exh. C-1).
On January 25, 1958, petitioners instituted the present action in the Court of
First Instance of Isabela against respondents, to quiet title to said portion
(19,964 square meters) formed by accretion, alleging in their complaint
(docketed as Civil Case No. 1171) that they and their predecessors-in-
74

interest, were formerly in peaceful and continuous possession thereof, until
September, 1948, when respondents entered upon the land under claim of
ownership. Petitioners also asked for damages corresponding to the value of
the fruits of the land as well as attorney's fees and costs. In their answer
(dated February 18, 1958), respondents claim ownership in themselves,
asserting that they have been in continuous, open, and undisturbed
possession of said portion, since prior to the year 1933 to the present.
After trial, the Court of First Instance of Isabela, on May 4, 1959, rendered a
decision adjudging the ownership of the portion in question to petitioners,
and ordering respondents to vacate the premises and deliver possession
thereof to petitioners, and to pay to the latter P250.00 as damages and
costs. Said decision, in part, reads:
It is admitted by the parties that the land involved in this action was
formed by the gradual deposit of alluvium brought about by the action
of the Cagayan River, a navigable river. We are inclined to believe that
the accretion was formed on the northeastern side of the land covered
by Original Certificate of Title No. 2982 after the survey of the
registered land in 1931, because the surveyors found out that the
northeastern boundary of the land surveyed by them was the Cagayan
River, and not the land in question. Which is indicative of the fact that
the accretion has not yet started or begun in 1931. And, as declared
by Pedro Laman, defendant witness and the boundary owner on the
northwest of the registered land of the plaintiffs, the accretion was a
little more than one hectare, including the stony portion, in 1940 or
1941. Therefore, the declarations of the defendant Domingo Calalung
and his witness, Vicente C. Bacani, to the effect that the land in
question was formed by accretion since 1933 do not only contradict
the testimony of defendants' witness Pedro Laman, but could not
overthrow the incontestable fact that the accretion with an area of 4
hectare more or less, was formed in 1948, reason for which, it was
only declared in that same year for taxation purposes by the
defendants under Tax Dec. No. 257 (Exh. "2") when they entered upon
the land. We could not give credence to defendants' assertion that Tax
Dec. No. 257 (Exh. "2") cancelled Tax Dee. No. 28226 (Exh. "1"),
because Exh. "2" says that "tax under this declaration begins with the
year 1948. But, the fact that defendants declared the land for taxation
purposes since 1948, does not mean that they become the owner of
the land by mere occupancy, for it is a new provision of the New Civil
Code that ownership of a piece of land cannot be acquired by
occupation (Art. 714, New Civil Code). The land in question being an
accretion to the mother or registered land of the plaintiffs, the
accretion belongs to the plaintiffs (Art. 457, New Civil Code; Art. 366,
75

Old Civil Code). Assuming arguendo, that the accretion has been
occupied by the defendants since 1948, or earlier, is of no moment,
because the law does not require any act of possession on the part of
the owner of the riparian owner, from the moment the deposit
becomes manifest (Roxas v. Tuason, 9 Phil. 408; Cortez v. City of
Manila, 10 Phil. 567). Further, no act of appropriation on the part of
the reparian owner is necessary, in order to acquire ownership of the
alluvial formation, as the law does not require the same (3 Manresa,
C.C., pp. 321-326).
This brings us now to the determination of whether the defendants,
granting that they have been in possession of the alluvium since 1948,
could have acquired the property by prescription. Assuming that they
occupied the land in September, 1948, but considering that the action
was commenced on January 25, 1958, they have not been in
possession of the land for ten (10) years; hence, they could not have
acquired the land by ordinary prescription (Arts. 1134 and 1138, New
Civil Code). Moreover, as the alluvium is, by law, part and parcel of
the registered property, the same may be considered as registered
property, within the meaning of Section 46 of Act No. 496: and,
therefore, it could not be acquired by prescription or adverse
possession by another person.
Unsatisfied, respondents appealed to the Court of Appeals, which rendered,
on September 14, 1960, the decision adverted to at the beginning of this
opinion, partly stating:
That the area in controversy has been formed through a gradual
process of alluvium, which started in the early thirties, is a fact
conclusively established by the evidence for both parties. By law,
therefore, unless some superior title has supervened, it should
properly belong to the riparian owners, specifically in accordance with
the rule of natural accession in Article 366 of the old Civil Code (now
Article 457), which provides that "to the owner of lands adjoining the
banks of rivers, belongs the accretion which they gradually receive
from the effects of the current of the waters." The defendants,
however, contend that they have acquired ownership through
prescription. This contention poses the real issue in this case. The
Courta quo, has resolved it in favor of the plaintiffs, on two grounds:
First, since by accession, the land in question pertains to the original
estate, and since in this instance the original estate is registered, the
accretion, consequently, falls within the purview of Section 46 of Act
No. 496, which states that "no title to registered land in derogation to
that of the registered owner shall be acquired by prescription or
76

adverse possession"; and, second, the adverse possession of the
defendant began only in the month of September, 1948, or less than
the 10-year period required for prescription before the present action
was instituted.
As a legal proposition, the first ground relied upon by the trial court, is
not quite correct. An accretion to registered land, while declared by
specific provision of the Civil Code to belong to the owner of the land
as a natural accession thereof, does not ipso jure become entitled to
the protection of the rule of imprescriptibility of title established by the
Land Registration Act. Such protection does not extend beyond the
area given and described in the certificate. To hold otherwise, would
be productive of confusion. It would virtually deprive the title, and the
technical description of the land given therein, of their character of
conclusiveness as to the identity and area of the land that is
registered. Just as the Supreme Court, albeit in a negative manner,
has stated that registration does not protect the riparian owner against
the erosion of the area of his land through gradual changes in the
course of the adjoining stream (Payatas Estate Development Co. v.
Tuason, 53 Phil. 55), so registration does not entitle him to all the
rights conferred by Land Registration Act, in so far as the area added
by accretion is concerned. What rights he has, are declared not by said
Act, but by the provisions of the Civil Code on accession: and these
provisions do not preclude acquisition of the addition area by another
person through prescription. This Court has held as much in the case
of Galindez, et al. v. Baguisa, et al., CA-G.R. No. 19249-R, July 17,
1959.
We now proposed to review the second ground relied upon by the trial
court, regarding the length of time that the defendants have been in
possession. Domingo Calalung testified that he occupied the land in
question for the first time in 1934, not in 1948 as claimed by the
plaintiffs. The area under occupancy gradually increased as the years
went by. In 1946, he declared the land for purposes of taxation
(Exhibit 1). This tax declaration was superseded in 1948 by another
(Exhibit 2), after the name of the municipality wherein it is located was
changed from Tumauini to Magsaysay. Calalung's testimony is
corroborated by two witnesses, both owners of properties nearby.
Pedro Laman, 72 years of age, who was Municipal president of
Tumauini for three terms, said that the land in question adjoins his
own on the south, and that since 1940 or 1951, he has always known
it to be in the peaceful possession of the defendants. Vicente C. Bacani
testified to the same effect, although, he said that the defendants'
77

possession started sometime in 1933 or 1934. The area thereof, he
said, was then less than one hectare.
We find the testimony of the said witnesses entitled to much greater
weight and credence than that of the plaintiff Pedro Grande and his
lone witness, Laureana Rodriguez. The first stated that the defendants
occupied the land in question only in 1948; that he called the latter's
attention to the fact that the land was his, but the defendants, in turn,
claimed that they were the owners, that the plaintiffs did not file an
action until 1958, because it was only then that they were able to
obtain the certificate of title from the surveyor, Domingo Parlan; and
that they never declared the land in question for taxation purposes or
paid the taxes thereon. Pedro Grande admitted that the defendants
had the said land surveyed in April, 1958, and that he tried to stop it,
not because he claimed the accretion for himself and his co-plaintiffs,
but because the survey included a portion of the property covered by
their title. This last fact is conceded by the defendants who,
accordingly, relinquished their possession to the part thus included,
containing an area of some 458 square meters.1wph1.t
The oral evidence for the defendants concerning the period of their
possession from 1933 to 1958 is not only preponderant in itself,
but is, moreover, supported by the fact that it is they and not the
plaintiffs who declared the disputed property for taxation, and by the
additional circumstance that if the plaintiff had really been in prior
possession and were deprived thereof in 1948, they would have
immediately taken steps to recover the same. The excuse they gave
for not doing so, namely, that they did not receive their copy of the
certificate of title to their property until 1958 for lack of funds to pay
the fees of the surveyor Domingo Parlan, is too flimsy to merit any
serious consideration. The payment of the surveyor's fees had nothing
to do with their right to obtain a copy of the certificate. Besides, it was
not necessary for them to have it in their hands, in order to file an
action to recover the land which was legally theirs by accession and of
which, as they allege, they had been illegally deprived by the
defendants. We are convinced, upon consideration of the evidence,
that the latter, were really in possession since 1934, immediately after
the process of alluvion started, and that the plaintiffs woke up to their
rights only when they received their copy of the title in 1958. By then,
however, prescription had already supervened in favor of the
defendants.
It is this decision of the Court of Appeals which petitioners seek to be
reviewed by us.
78

The sole issue for resolution in this case is whether respondents have
acquired the alluvial property in question through prescription.
There can be no dispute that both under Article 457 of the New Civil Code
and Article 366 of the old, petitioners are the lawful owners of said alluvial
property, as they are the registered owners of the land which it adjoins. The
question is whether the accretion becomes automatically registered land just
because the lot which receives it is covered by a Torrens title thereby
making the alluvial property imprescriptible. We agree with the Court of
Appeals that it does not, just as an unregistered land purchased by the
registered owner of the adjoining land does not, by extension, become ipso
facto registered land. Ownership of a piece of land is one thing, and
registration under the Torrens system of that ownership is quite another.
Ownership over the accretion received by the land adjoining a river is
governed by the Civil Code. Imprescriptibility of registered land is provided
in the registration law. Registration under the Land Registration and
Cadastral Acts does not vest or give title to the land, but merely confirms
and thereafter protects the title already possessed by the owner, making it
imprescriptible by occupation of third parties. But to obtain this protection,
the land must be placed under the operation of the registration laws wherein
certain judicial procedures have been provided. The fact remain, however,
that petitioners never sought registration of said alluvial property (which was
formed sometime after petitioners' property covered by Original Certificate
of Title No. 2982 was registered on June 9, 1934) up to the time they
instituted the present action in the Court of First Instance of Isabela in 1958.
The increment, therefore, never became registered property, and hence is
not entitled or subject to the protection of imprescriptibility enjoyed by
registered property under the Torrens system. Consequently, it was subject
to acquisition through prescription by third persons.
The next issue is, did respondents acquire said alluvial property through
acquisitive prescription? This is a question which requires determination of
facts: physical possession and dates or duration of such possession. The
Court of Appeals, after analyzing the evidence, found that respondents-
appellees were in possession of the alluvial lot since 1933 or 1934, openly,
continuously and adversely, under a claim of ownership up to the filing of
the action in 1958. This finding of the existence of these facts, arrived at by
the Court of Appeals after an examination of the evidence presented by the
parties, is conclusive as to them and can not be reviewed by us.
The law on prescription applicable to the case is that provided in Act 190 and
not the provisions of the Civil Code, since the possession started in 1933 or
1934 when the pertinent articles of the old Civil Code were not in force and
before the effectivity of the new Civil Code in 1950. Hence, the conclusion of
79

the Court of Appeals that the respondents acquired alluvial lot in question by
acquisitive prescription is in accordance with law.
The decision of the Court of Appeals under review is hereby affirmed, with
costs against the petitioners. So ordered.
Bengzon, C.J., Padilla, Bautista Angelo, Labrador, Concepcion, Paredes and
Dizon, JJ., concur.
Reyes, J.B.L., Regala and Makalintal, JJ., took no part.
SIMPLICIO BINALAY, PONCIANO GANNABAN, NICANOR MACUTAY,
DOMINGO ROSALES, GREGORIO ARGONZA, EUSTAQUIO BAUA,
FLORENTINO ROSALES, TEODORO MABBORANG, PATRICIO
MABBORANG and FULGENCIO MORA, petitioners
vs.
GUILLERMO MANALO and COURT OF APPEALS, respondents.
Josefin De Alban Law Office for Petitioners.

FELICIANO, J.:p
The late Judge Taccad originally owned a parcel of land situated in Tumauini,
Isabela having an estimated area of twenty (20) hectares. The western
portion of this land bordering on the Cagayan River has an elevation lower
than that of the eastern portion which borders on the national road. Through
the years, the western portion would periodically go under the waters of the
Cagayan River as those waters swelled with the coming of the rains. The
submerged portion, however, would re-appear during the dry season from
January to August. It would remain under water for the rest of the year, that
is, from September to December during the rainy season.
The ownership of the landholding eventually moved from one person to
another. On 9 May 1959, respondent Guillermo Manalo acquired 8.65
hectares thereof from Faustina Taccad, daughter of Judge Juan Taccad. The
land sold was described in the Deed of Absolute Sale
1
as follows:
. . . a parcel of agricultural land in Balug, Tumauini, Isabela,
containing an area of 8.6500 hectares, more or less; bounded on
the North by Francisco Forto on the East by National Road; on
South by Julian Tumolva and on the West by Cagayan River;
declared for taxation under Tax Declaration No. 12681 in the
name of Faustina Taccad, and assessed at P 750.00. . . .
80

Later in 1964, respondent Manalo purchased another 1.80 hectares from
Gregorio Taguba who had earlier acquired the same from Judge Juan
Taccad. The second purchase brought the total acquisition of respondent
Manalo to 10.45 hectares. The second piece of property was more
particularly described as follows:
. . . a piece of agricultural land consisting of tobacco land, and
containing an area of 18,000 square meters, more or less,
bounded on the North by Balug Creek; on the South, by Faustina
Taccad (now Guillermo R. Manalo); on the East, by a Provincial
Road; and on the West, by Cagayan River assessed at P 440.00,
as tax Declaration No. 3152. . . .
2

During the cadastral survey conducted at Balug, Tumauini, Isabela on 21
October 1969, the two (2) parcels of land belonging to respondent Manalo
were surveyed and consolidated into one lot, designated as Lot No. 307, Pls-
964. Lot 307 which contains 4.6489 hectares includes: (a) the whole of the
1.80 hectares acquired from Gregorio Taguba; and (b) 2.8489 hectares out
of the 8.65 hectares purchased from Faustina Taccad. As the survey was
conducted on a rainy month, a portion of the land bought from Faustina
Taccad then under water was left unsurveyed and was not included in Lot
307.
The Sketch Plan
3
submitted during the trial of this case and which was
identified by respondent Manalo shows that the Cagayan River running from
south to north, forks at a certain point to form two (2) branchesthe
western and the eastern branchesand then unites at the other end, further
north, to form a narrow strip of land. The eastern branch of the river cuts
through the land of respondent Manalo and is inundated with water only
during the rainy season. The bed of the eastern branch is the submerged or
the unsurveyed portion of the land belonging to respondent Manalo. For
about eight (8) months of the year when the level of water at the point
where the Cagayan River forks is at its ordinary depth, river water does not
flow into the eastern branch. While this condition persists, the eastern bed is
dry and is susceptible to cultivation.
Considering that water flowed through the eastern branch of the Cagayan
River when the cadastral survey was conducted, the elongated strip of land
formed by the western and the eastern branches of the Cagayan River
looked very much like an island. This strip of land was surveyed on 12
December 1969.
4
It was found to have a total area of 22.7209 hectares and
was designated as Lot 821 and Lot 822. The area of Lot 822 is 10.8122
hectares while Lot 821 has an area of 11.9087 hectares. Lot 821 is located
directly opposite Lot 307 and is separated from the latter only by the eastern
81

branch of the Cagayan River during the rainy season and, during the dry
season, by the exposed, dry river bed, being a portion of the land bought
from Faustina Taccad. Respondent Manalo claims that Lot 821 also belongs
to him by way of accretion to the submerged portion of the property to
which it is adjacent.
Petitioners who are in possession of Lot 821, upon the other hand, insist that
they own Lot 821. They occupy the outer edges of Lot 821 along the river
banks, i.e., the fertile portions on which they plant tobacco and other
agricultural products. They also cultivate the western strip of the unsurveyed
portion during summer.
5
This situation compelled respondent Manalo to file
a case for forcible entry against petitioners on 20 May 1969. The case was
dismissed by the Municipal Court of Tumauini, Isabela for failure of both
parties to appear. On 15 December 1972, respondent Manalo again filed a
case for forcible entry against petitioners. The latter case was similarly
dismissed for lack of jurisdiction by the Municipal Court of Tumauini, Isabela.
On 24 July 1974, respondent Manalo filed a complaints
6
before the then
Court of First Instance of Isabela, Branch 3 for quieting of title, possession
and damages against petitioners. He alleged ownership of the two (2)
parcels of land he bought separately from Faustina Taccad and Gregorio
Taguba for which reason he prayed that judgment be entered ordering
petitioners to vacate the western strip of the unsurveyed portion.
Respondent Manalo likewise prayed that judgment be entered declaring him
as owner of Lot 821 on which he had laid his claim during the survey.
Petitioners filed their answer denying the material allegations of the
complaint. The case was then set for trial for failure of the parties to reach
an amicable agreement or to enter into a stipulation of facts.
7
On 10
November 1982, the trial court rendered a decision with the following
dispositive portion:
WHEREFORE, in the light of the foregoing premises, the Court
renders judgment against the defendants and in favor of the
plaintiff and orders:
1. That plaintiff, Guillermo Manalo, is declared the lawful owner
of the land in question, Lot No. 821, Pls-964 of Tumauini
Cadastre, and which is more particularly described in paragraph
2-b of the Complaint;
2. That the defendants are hereby ordered to vacate the
premises of the land in question, Lot No. 821, Pls-964 of
82

Tumauini Cadastre, and which is more particularly described in
paragraph 2-b of the Complaint;
3. That the defendants are being restrained from entering the
premises of the land in question, Lot No. 821, Pls-964 of
Tumauini Cadastre, and which is more particularly described in
paragraph 2-b of the Complaint; and
4. That there is no pronouncement as to attorney's fees and
costs.
SO ORDERED.
8

Petitioners appealed to the Court of Appeals which, however, affirmed the
decision of the trial court. They filed a motion for reconsideration, without
success.
While petitioners insist that Lot 821 is part of an island surrounded by the
two (2) branches of the Cagayan River, the Court of Appeals found
otherwise. The Court of Appeals concurred with the finding of the trial court
that Lot 821 cannot be considered separate and distinct from Lot 307 since
the eastern branch of the Cagayan River substantially dries up for the most
part of the year such that when this happens, Lot 821 becomes physically
(i.e., by land) connected with the dried up bed owned by respondent
Manalo. Both courts below in effect rejected the assertion of petitioners that
the depression on the earth's surface which separates Lot 307 and Lot 821
is, during part of the year, the bed of the eastern branch of the Cagayan
River.
It is a familiar rule that the findings of facts of the trial court are entitled to
great respect, and that they carry even more weight when affirmed by the
Court of Appeals.
9
This is in recognition of the peculiar advantage on the
part of the trial court of being able to observe first-hand the deportment of
the witnesses while testifying. Jurisprudence is likewise settled that the
Court of Appeals is the final arbiter of questions of fact.
10
But whether a
conclusion drawn from such findings of facts is correct, is a question of law
cognizable by this Court.
11

In the instant case, the conclusion reached by both courts below apparently
collides with their findings that periodically at the onset of and during the
rainy season, river water flows through the eastern bed of the Cagayan
River. The trial court held:
83

The Court believes that the land in controversy is of the nature
and character of alluvion (Accretion), for it appears that during
the dry season, the body of water separating the same land in
controversy (Lot No. 821, Pls-964) and the two (2) parcels of
land which the plaintiff purchased from Gregorio Taguba and
Justina Taccad Cayaba becomes a marshy land and is only six
(6) inches deep and twelve (12) meters in width at its widest in
the northern tip (Exhs. "W", "W-l", "W-2", "W-3" and "W-4"), It
has been held by our Supreme Court that "the owner of the
riparian land which receives the gradual deposits of alluvion,
does not have to make an express act of possession. The law
does not require it, and the deposit created by the current of the
water becomes manifest" (Roxas vs. Tuazon, 6 Phil. 408).
12

The Court of Appeals adhered substantially to the conclusion reached by the
trial court, thus:
As found by the trial court, the disputed property is not an island
in the strict sense of the word since the eastern portion of the
said property claimed by appellants to be part of the Cagayan
River dries up during summer. Admittedly, it is the action of the
heavy rains which comes during rainy season especially from
September to November which increases the water level of the
Cagayan river. As the river becomes swollen due to heavy rains,
the lower portion of the said strip of land located at its
southernmost point would be inundated with water. This is
where the water of the Cagayan river gains its entry.
Consequently, if the water level is high the whole strip of land
would be under water.
In Government of the Philippine Islands vs. Colegio de San Jose, it was held
that
According to the foregoing definition of the words
"ordinary" and "extra-ordinary," the highest depth of
the waters of Laguna de Bay during the dry season is
the ordinary one, and the highest depth they attain
during the extra-ordinary one (sic); inasmuch as the
former is the one which is regular, common, natural,
which occurs always or most of the time during the
year, while the latter is uncommon, transcends the
general rule, order and measure, and goes beyond
that which is the ordinary depth. If according to the
definition given by Article 74 of the Law of Waters
84

quoted above, the natural bed or basin of the lakes
is the ground covered by their waters when at their
highest ordinary depth, the natural bed or basin of
Laguna de Bay is the ground covered by its waters
when at their highest depth during the dry season,
that is up to the northeastern boundary of the two
parcels of land in question.
We find the foregoing ruling to be analogous to the case at bar.
The highest ordinary level of the waters of the Cagayan River is
that attained during the dry season which is confined only on the
west side of Lot [821] and Lot [822]. This is the natural Cagayan
river itself. The small residual of water between Lot [821] and
307 is part of the small stream already in existence when the
whole of the late Judge Juan Taccad's property was still
susceptible to cultivation and uneroded.
13

The Court is unable to agree with the Court of Appeals that Government of
the Philippine Islands vs. Colegio de San Jose
14
is applicable to the present
case. That case involved Laguna de Bay; since Laguna de Bay is a lake, the
Court applied the legal provisions governing the ownership and use of lakes
and their beds and shores, in order to determine the character and
ownership of the disputed property. Specifically, the Court applied the
definition of the natural bed or basin of lakes found in Article 74 of the Law
of Waters of 3 August 1866. Upon the other hand, what is involved in the
instant case is the eastern bed of the Cagayan River.
We believe and so hold that Article 70 of the Law of Waters of 3 August
1866 is the law applicable to the case at bar:
Art. 70. The natural bed or channel of a creek or river is the
ground covered by its waters during the highest floods.
(Emphasis supplied)
We note that Article 70 defines the natural bed or channel of a creek
or river as the ground covered by its waters during the highest floods.
The highest floods in the eastern branch of the Cagayan River occur
with the annual coming of the rains as the river waters in their onward
course cover the entire depressed portion. Though the eastern bed
substantially dries up for the most part of the year (i.e., from January
to August), we cannot ignore the periodical swelling of the waters
( i.e., from September to December) causing the eastern bed to be
covered with flowing river waters.
85

The conclusion of this Court that the depressed portion is a river bed rests
upon evidence of record. Firstly, respondent Manalo admitted in open court
that the entire area he bought from Gregorio Taguba was included in Lot
307.
15
If the 1.80 hectares purchased from Gregorio Taguba was included in
Lot 307, then the Cagayan River referred to as the western boundary in the
Deed of Sale transferring the land from Gregorio Taguba to respondent
Manalo as well as the Deed of Sale signed by Faustina Taccad, must refer to
the dried up bed (during the dry months) or the eastern branch of the river
(during the rainy months). In the Sketch Plan attached to the records of the
case, Lot 307 is separated from the western branch of the Cagayan River by
a large tract of land which includes not only Lot 821 but also what this Court
characterizes as the eastern branch of the Cagayan River.
Secondly, the pictures identified by respondent Manalo during his direct
examination depict the depressed portion as a river bed. The pictures,
marked as Exhibits "W" to "W-4", were taken in July 1973 or at a time when
the eastern bed becomes visible.
16
Thus, Exhibit "W-2" which according to
respondent Manalo was taken facing the east and Exhibit "W-3" which was
taken facing the west both show that the visible, dried up portion has a
markedly lower elevation than Lot 307 and Lot 821. It has dike-like slopes
on both sides connecting it to Lot 307 and Lot 821 that are vertical upward
and very prominent. This topographic feature is compatible with the fact that
a huge volume of water passes through the eastern bed regularly during the
rainy season. In addition, petitioner Ponciano Gannaban testified that one
had to go down what he called a "cliff" from the surveyed portion of the land
of respondent Manalo to the depressed portion. The cliff, as related by
petitioner Gannaban, has a height of eight (8) meters.
17

The records do not show when the Cagayan River began to carve its eastern
channel on the surface of the earth. However, Exhibit "E"
18
for the
prosecution which was the Declaration of Real Property standing in the name
of Faustina Taccad indicates that the eastern bed already existed even
before the sale to respondent Manalo. The words "old bed" enclosed in
parenthesesperhaps written to make legitimate the claim of private
ownership over the submerged portionis an implied admission of the
existence of the river bed. In the Declaration of Real Property made by
respondent Manalo, the depressed portion assumed the name Rio Muerte de
Cagayan. Indeed, the steep dike-like slopes on either side of the eastern bed
could have been formed only after a prolonged period of time.
Now, then, pursuant to Article 420 of the Civil Code, respondent Manalo did
not acquire private ownership of the bed of the eastern branch of the river
even if it was included in the deeds of absolute sale executed by Gregorio
Taguba and Faustina Taccad in his favor. These vendors could not have
86

validly sold land that constituted property of public dominion. Article 420 of
the Civil Code states:
The following things are property of public dominion:
(1) Those intended for public use, such as roads,
canals, rivers, torrents, ports and bridges constructed by the
State, banks, shores, roadsteads, and others of similar
character;
(2) Those which belong to the State, without being for public
use, and are intended for some public service or for the
development of the national wealth. (Emphasis supplied)
Although Article 420 speaks only of rivers and banks, "rivers" is a composite
term which includes: (1) the running waters, (2) the bed, and (3) the
banks.
19
Manresa, in commenting upon Article 339 of the Spanish Civil Code
of 1889 from which Article 420 of the Philippine Civil Code was taken,
stressed the public ownership of river beds:
La naturaleza especial de los rios, en punto a su disfrute general,
hace que sea necesario considerar en su relacion de dominio
algo mas que sus aguas corrientes. En efecto en todo rio es
preciso distinguir 1. esta agua corriente; 2. el alveo o cauce,
y 3. las riberas. Ahora bien: son estas dos ultimas cosas siempre
de dominio publico, como las aguas?
Realmente no puede imaginarse un rio sin alveo y sin ribera; de
suerte que al decir el Codigo civil que los rios son de dominio
publico, parece que debe ir implicito el dominio publico de
aquellos tres elementos que integran el rio. Por otra parte, en
cuanto a los alveos o cauces tenemos la declaracion del art. 407,
num 1, donde dice: son de dominion publico . . . los rios y sus
cauces naturales; declaracion que concuerda con lo que dispone
el art. 34 de la ley de [Aguas], segun el cual, son de dominion
publico: 1. los alveos o cauces de los arroyos que no se hallen
comprendidos en el art. 33, y 2. los alveos o cauces naturales de
los rios en la extension que cubran sus aguas en las mayores
crecidas ordinarias.
20
(Emphasis supplied)
The claim of ownership of respondent Manalo over the submerged portion is
bereft of basis even if it were alleged and proved that the Cagayan River
first began to encroach on his property after the purchase from Gregorio
Taguba and Faustina Taccad. Article 462 of the Civil Code would then apply
87

divesting, by operation of law, respondent Manalo of private ownership over
the new river bed. The intrusion of the eastern branch of the Cagayan River
into his landholding obviously prejudiced respondent Manalo but this is a
common occurrence since estates bordering on rivers are exposed to floods
and other evils produced by the destructive force of the waters. That loss is
compensated by, inter alia, the right of accretion acknowledged by Article
457 of the Civil Code.
21
It so happened that instead of increasing the size of
Lot 307, the eastern branch of the Cagayan River had carved a channel on
it.
We turn next to the issue of accretion. After examining the records of the
case, the Court considers that there was no evidence to prove that Lot 821
is an increment to Lot 307 and the bed of the eastern branch of the river.
Accretion as a mode of acquiring property under Article 457 of the Civil Code
requires the concurrence of three (3) requisites: (a) that the deposition of
soil or sediment be gradual and imperceptible; (b) that it be the result of the
action of the waters of the river (or sea); and (c) that the land where
accretion takes place is adjacent to the banks of rivers (or the sea
coast).
22
The Court notes that the parcels of land bought by respondent
Manalo border on the eastern branch of the Cagayan River. Any accretion
formed by this eastern branch which respondent Manalo may claim must be
deposited on or attached to Lot 307. As it is, the claimed accretion (Lot 821)
lies on the bank of the rivernot adjacent to Lot 307 but directly opposite Lot
307 across the river.
Assuming (arguendo only) that the Cagayan River referred to in the Deeds
of Sale transferring ownership of the land to respondent Manalo is the
western branch, the decision of the Court of Appeals and of the trial court
are bare of factual findings to the effect that the land purchased by
respondent Manalo received alluvium from the action of the aver in a slow
and gradual manner. On the contrary, the decision of the lower court made
mention of several floods that caused the land to reappear making it
susceptible to cultivation. A sudden and forceful action like that of flooding is
hardly the alluvial process contemplated under Article 457 of the Civil Code.
It is the slow and hardly perceptible accumulation of soil deposits that the
law grants to the riparian owner.
Besides, it is important to note that Lot 821 has an area of 11.91 hectares.
Lot 821 is the northern portion of the strip of land having a total area of
22.72 hectares. We find it difficult to suppose that such a sizable area as Lot
821 resulted from slow accretion to another lot of almost equal size. The
total landholding purchased by respondent Manalo is 10.45 hectares (8.65
hectares from Faustina Taccad and 1.80 hectares from Gregorio Taguba in
1959 and 1964, respectively), in fact even smaller than Lot 821 which he
88

claims by way of accretion. The cadastral survey showing that Lot 821 has
an area of 11.91 hectares was conducted in 1969. If respondent Manalo's
contention were accepted, it would mean that in a span of only ten (10)
years, he had more than doubled his landholding by what the Court of
Appeals and the trial court considered as accretion. As already noted, there
are steep vertical dike-like slopes separating the depressed portion or river
bed and Lot 821 and Lot 307. This topography of the land, among other
things, precludes a reasonable conclusion that Lot 821 is an increment to the
depressed portion by reason of the slow and constant action of the waters of
either the western or the eastern branches of the Cagayan River.
We turn finally to the issue of ownership of Lot 821. Respondent Manalo's
claim over Lot 821 rests on accretion coupled with alleged prior possession.
He alleged that the parcels of land he bought separately from Gregorio
Taguba and Faustina Taccad were formerly owned by Judge Juan Taccad
who was in possession thereof through his (Judge Taccad's) tenants. When
ownership was transferred to him, respondent Manalo took over the
cultivation of the property and had it declared for taxation purposes in his
name. When petitioners forcibly entered into his property, he twice instituted
the appropriate action before the Municipal Trial Court of Tumauini, Isabela.
Against respondent Manalo's allegation of prior possession, petitioners
presented tax declarations standing in their respective names. They claimed
lawful, peaceful and adverse possession of Lot 821 since 1955.
If respondent Manalo had proved prior possession, it was limited physically
to Lot 307 and the depressed portion or the eastern river bed. The testimony
of Dominga Malana who was a tenant for Justina Taccad did not indicate that
she was also cultivating Lot 821. In fact, the complaints for forcible entry
lodged before the Municipal Trial Court of Tumauini, Isabela pertained only
to Lot 307 and the depressed portion or river bed and not to Lot 821. In the
same manner, the tax declarations presented by petitioners conflict with
those of respondent Manalo. Under Article 477 of the Civil Code, the plaintiff
in an action for quieting of title must at least have equitable title to or
interest in the real property which is the subject matter of the action. The
evidence of record on this point is less than satisfactory and the Court feels
compelled to refrain from determining the ownership and possession of Lot
821, adjudging neither petitioners nor respondent Manalo as owner(s)
thereof.
WHEREFORE, the Decision and Resolution of the Court of Appeals in CA-GR
CV No. 04892 are hereby SET ASIDE. Respondent Manalo is hereby declared
the owner of Lot 307. The regularly submerged portion or the eastern bed of
the Cagayan River is hereby DECLARED to be property of public dominion.
The ownership of Lot 821 shall be determined in an appropriate action that
89

may be instituted by the interested parties inter se. No pronouncement as to
costs.
SO ORDERED.
Fernan, C.J., Gutierrez, Jr., Bidin and Davide, Jr., JJ., concur.

You might also like